75
Physical Sciences Time: 100 Minutes Questions 1–77 DO NO T BEGIN THIS SECTION UNTIL Y OU ARE T OLD T O DO SO . 03 MCAT FL Test3 06/27/2003 10:25 AM Page 1

Full length 3

Embed Size (px)

DESCRIPTION

MCAT Practice

Citation preview

Page 1: Full length 3

Physical SciencesTime: 100 Minutes

Questions 1–77

DO NOT BEGIN THIS SECTION UNTIL YOU ARE TOLD TO DO SO.

03 MCAT FL Test3 06/27/2003 10:25 AM Page 1

Page 2: Full length 3

2

GO ON TO THE NEXT PAGE.

PHYSICAL SCIENCES

DIRECTIONS: Most of the questions in the PhysicalSciences test are organized into groups, with adescriptive passage preceding each group of ques-tions. Study the passage, then select the single bestanswer to each question in the group. Some of thequestions are not based on a descriptive passage; youmust also select the best answer to these questions. Ifyou are unsure of the best answer, eliminate thechoices that you know are incorrect, then select ananswer from the choices that remain. Indicate yourselection by blackening the corresponding circle onyour answer sheet. A periodic table is provided belowfor your use with the questions.

1

H

1.0

2

He

4.0

3

Li

6.9

4

Be

9.0

5

B

10.8

6

C

12.0

7

N

14.0

8

O

16.0

9

F

19.0

10

Ne

20.2

11

Na

23.0

12

Mg

24.3

13

Al

27.0

14

Si

28.1

15

P

31.0

16

S

32.1

17

Cl

35.5

18

Ar

39.9

19

K

39.1

20

Ca

40.1

21

Sc

45.0

22

Ti

47.9

23

V

50.9

24

Cr

52.0

25

Mn

54.9

26

Fe

55.8

27

Co

58.9

28

Ni

58.7

29

Cu

63.5

30

Zn

65.4

31

Ga

69.7

32

Ge

72.6

33

As

74.9

34

Se

79.0

35

Br

79.9

36

Kr

83.8

37

Rb

85.5

38

Sr

87.6

39

Y

88.9

40

Zr

91.2

41

Nb

92.9

42

Mo

95.9

43

Tc

(98)

44

Ru

101.1

45

Rh

102.9

46

Pd

106.4

47

Ag

107.9

48

Cd

112.4

49

In

114.8

50

Sn

118.7

51

Sb

121.8

52

Te

127.6

53

I

126.9

54

Xe

131.3

55

Cs

132.9

56

Ba

137.3

57

La *

138.9

72

Hf

178.5

73

Ta

180.9

74

W

183.9

75

Re

186.2

76

Os

190.2

77

Ir

192.2

78

Pt

195.1

79

Au

197.0

80

Hg

200.6

81

Tl

204.4

82

Pb

207.2

83

Bi

209.0

84

Po

(209)

85

At

(210)

86

Rn

(222)

87

Fr

(223)

88

Ra

226.0

89

Ac †

227.0

104

Rf

(261)

105

Ha

(262)

106

Unh

(263)

107

Uns

(262)

108

Uno

(265)

109

Une

(267)

*

58

Ce

140.1

59

Pr

140.9

60

Nd

144.2

61

Pm

(145)

62

Sm

150.4

63

Eu

152.0

64

Gd

157.3

65

Tb

158.9

66

Dy

162.5

67

Ho

164.9

68

Er

167.3

69

Tm

168.9

70

Yb

173.0

71

Lu

175.0

90

Th

232.0

91

Pa

(231)

92

U

238.0

93

Np

(237)

94

Pu

(244)

95

Am

(243)

96

Cm

(247)

97

Bk

(247)

98

Cf

(251)

99

Es

(252)

100

Fm

(257)

101

Md

(258)

102

No

(259)

103

Lr

(260)

PERIODIC TABLE OF THE ELEMENTS

03 MCAT FL Test3 06/27/2003 10:25 AM Page 2

Page 3: Full length 3

Passage I (Questions 1–6)

Historically, two different methods have been used toestimate the fluid pressure in capillary beds.

Method 1

A glass pipette is inserted into the capillary. The levelof blood rising in the pipette is measured and used to cal-culate the pressure. Alternatively, an inert fluid of densityρ can be placed in the pipette and its height h can be mea-sured. The pressure in the capillary is given by ρgh, whereg is the acceleration due to gravity.

Figure 1

Method 2

The pressure can be measured indirectly in the follow-ing way. A section of gut tissue is removed from a speci-men and placed on a beam balance. Blood is circulatedthrough the tissue by a pump. The arterial pressure is thendecreased. This leads to a decrease in the capillary hydro-static pressure in the gut capillaries. The constant osmoticpressure of plasma proteins in the capillary causes absorp-tion of fluid from the gut section which will decrease itsweight. To prevent a change in the weight of the gut sec-tion, the venous pressure is increased. This tends toincrease the capillary pressure, reducing the flow of fluidfrom the gut tissue into the capillaries. The capillary pres-sure is thus held constant (and the balance kept level) asthe arterial pressure is decreased and the venous pressureincreased. The arterial and venous pressures meet at thecapillary pressure being measured.

(Π = MRT, where Π is the osmotic pressure, M themolarity of the solutes, R the universal gas constant, and Tthe temperature in Kelvin.)

Figure 2

1. An arteriole is almost completely blocked by a bloodclot as shown below. At which point will the velocityof blood flow be the greatest? (Assume ideal laminarflow throughout.)

A. A

B. B

C. C

D. D

2. Assume that a mass (m) of 0.2 kg is placed 25 cm tothe right of the fulcrum. A section of gut, initiallyweighing 0.1 kg is placed 50 cm to the left of the ful-crum. During the experiment, the mass m is seen todescend. In order to maintain the balance level thefollowing action should be taken:

A. the mass m should be moved away from the ful-crum.

B. the arterial pressure should be decreased.

C. the venous pressure should be increased.

D. an inert fluid with higher density than that ofblood should be used in the pipette.

A B C

Dclot

massmarterial

pressurevenouspressure

gut tissue

pipette

height of fluid

capillary bed

3

GO ON TO THE NEXT PAGE.

03 MCAT FL Test3 06/27/2003 10:25 AM Page 3

Page 4: Full length 3

3. A researcher using Method 1 to determine the capil-lary pressure fills the pipette with an inert fluid lessdense than blood. Compared to blood, the height ofthis fluid in the pipette will be:

A. higher because the fluid is less dense.

B. lower because the fluid is less dense.

C. the same because the pressure being measuredis the same.

D. the same because the velocity of blood flow inthe capillary bed is the same.

4. Which of the following is sufficient information todetermine the osmotic pressure of a solution?(Assume that the universal gas constant R is known.)

A. Mass of solid dissolved, volume of solvent,temperature

B. Molecular formula of solute, mass of solvent

C. Number of particles in solution, volume of solu-tion, temperature

D. Number of moles of solid dissolved, density ofsolution, temperature

5. Method 2 relies on keeping the beam balance level.Which of the following must be true if the balance islevel?

A. The arterial pressure equals the osmotic pres-sure.

B. The weight of the gut equals the weight of themass (m).

C. The net force on the beam is zero.

D. The net torque on the beam is zero.

6. Assume that the beam balance of Method 2 is ini-tially level. If the arterial pressure is decreased to alower level while everything else is held constant,which graph best represents the change in the mass ofthe gut following the decrease in arterial pressure?

A. C.

B. D.

mas

s of

gut

time

mas

s of

gut

time

mas

s of

gut

time

mas

s of

gut

time

4

GO ON TO THE NEXT PAGE.

03 MCAT FL Test3 06/27/2003 10:25 AM Page 4

Page 5: Full length 3

Passage II (Questions 7–12)

The automobile airbag was designed to inflate uponimpact and decrease the risk of injury to drivers and passen-gers. Among the challenges to its development was the needto find a reliable inflation mechanism that was sufficientlyrapid, controllable, and nontoxic. Prototypes employingcompressed gases failed to meet these criteria. Researchersthus turned their attention to chemical alternatives.

The ideal inflatant requires a chemical reaction in whichthe reactants are stable and relatively dense in the condensedphase while the products are mostly or completely gaseousat ambient temperature and pressure. Additionally, the idealchemical reaction would require a low activation energy andhave a high kinetic rate constant, without the large exother-micity characteristic of most such reactions. Traditionalexplosives such as nitroglycerin, C3H5N3O9 (l), wererejected almost immediately because of the extremelyexothermic nature of their conversion. Benign solids such ascalcium carbonate, CaCO3, were similarly rejected, becauseof their large activation requirements.

The desired attributes were finally found in sodiumazide, NaN3, a stable, dense, ionic solid which rapidlydecomposes into elemental sodium and nitrogen gas whenignited by an electrical impulse.

2NaN3 2Na + 3N2

Reaction 1

The gas generating mixture includes excess KNO3which reacts with the sodium metal from Reaction 1 toproduce additional N2 and potassium and sodium oxides(Reactions 2 and 3). These oxides react with SiO2 to pro-duce a non-toxic and stable alkaline silica (glass).

10Na + 2KNO3 K2O + 5Na2O + N2

Reaction 2

K2O + Na2O + SiO2 glass

Reaction 3

(R = 0.082�mL•

oalt•mK

�)

7. In order for the decomposition reaction to spreadthroughout the sodium azide after ignition, the ∆Hand ∆G for Reaction 1 must be, respectively:

A. positive, negative

B. positive, positive

C. negative, negative

D. negative, positive

8. A sodium azide air bag inflates to a volume of 45Liters at STP. According to the information containedin the passage, what is the mass of NaN3(Mol. Wt. = 65) that is required to inflate the bag?

A. 76 grams

B. 81 grams

C. 87 grams

D. 130 grams

9. Potassium chlorate, KClO3 (s), decomposes whenheated, yet it is unsuitable as an airbag inflatant. Allof the following characteristics of KClO3 make it apoor candidate for an air bag inflatant EXCEPT:

A. the decomposition requires a steady supply ofenergy due to its high activation energy and lowexothermicity.

B. the oxygen gas formed upon decomposition cre-ates a combustion hazard during an automobileaccident.

C. the potassium chloride formed upon decomposi-tion is a dense solid.

D. the oxygen gas formed upon decompositionleads to rapid expansion of the reaction mixture.

5

GO ON TO THE NEXT PAGE.

03 MCAT FL Test3 06/27/2003 10:25 AM Page 5

Page 6: Full length 3

10. Decomposition of which of the following transitionmetal complexes would produce the highest theoret-ical yield of carbon monoxide per gram of reactant?

A. Cr(CO)6B. Mn(CO)5C. Mo(CO)4D. Pd(CO)6

11. All of the following are resonance structures of N3–

EXCEPT:

A. C.

B. D.

12. A researcher wishes to make the decomposition ofsodium azide (Reaction 1) less favorable. Which ofthe following adjustments to the reaction would NOTdrive it to the left?

A. removal of NaN3 from the reaction mixture

B. addition of nitrogen gas to the reaction mixture

C. decreasing the pressure of the reaction mixture

D. increasing the temperature

N N N2-+ +

N N N+ --

NNN+

2-

N N N+

2-

6

GO ON TO THE NEXT PAGE.

03 MCAT FL Test3 06/27/2003 10:25 AM Page 6

Page 7: Full length 3

Passage III (Questions 13–17)

Fireworks have been used for centuries in celebrationsaround the world. One of the primary components of thesedevices, black powder, was developed by the Chinese overa thousand years ago and is still used today as a propellantand explosive. Black powder is composed of potassiumnitrate (KNO3), charcoal (primarily C) and sulfur (S8) in a75:15:10 ratio by weight. It is very stable if kept dry butcan easily be ignited by a spark or burning fuse to undergothe following reaction:

2KNO3 + 3C + S N2 + 3CO2 + K2S

Reaction 1

The basic firework is shown in Figure 1. Fireworksrely on a particular kind of combustion in which oxygen issupplied by oxidizing agents included in the pyrotechnicmixture. When ignited, the solid propellant begins to liq-uefy and vaporize allowing the fuel and oxidizing agentsto interact more intimately leading to rapid expansion ofgases. Delay fuses time the ignition of the other compart-ments to occur when the shell is high above ground.

Figure 1

The light generating units of the firework are calledstars and are dispersed and ignited by the bursting chargein each compartment. The intense colors of modern fire-works are generated by molecular emitters. For example,barium chloride emits green light (510–530 nm) and stron-tium chloride emits vibrant red light (605–650 nm). Manyof the molecular emitters are unstable at room temperatureand so cannot be placed directly into the firework. Instead,they are synthesized in the flame of the pyrotechnic reac-tion and exist for a short time before decomposing. Theflame temperature must be carefully adjusted so that theseemitters do not decompose too rapidly.

(h = 6.63 × 10–34 J•s, c = 3.0 × 108 ms–1)

13. The molar ratio of N to C to S in black powder isapproximately:

A. 75:125:35

B. 85:10:210

C. 75:15:10

D. 100:20:7

14. Which of the following chemicals is oxidized in thepyrotechnic reaction of gunpowder (Reaction 1)?

A. KNO3B. C

C. S

D. CO2

15. Which of the following bonds has the greatest ioniccharacter?

A. Ba–Cl

B. Sr–Cl

C. N–N

D. O=CO

16. Flares, a particular kind of pyrotechnic device, canburn underwater. Most materials, like wood, cannotburn underwater. Which of the following providesthe best explanation for this difference?

A. The combustion of wood has a lower ∆H.

B. Flares have a lower ignition temperature andcan be easily ignited by a spark or fuse.

C. Combustion of wood requires oxygen which isnot provided by water.

D. Water is a powerful flame retardant that extin-guishes flames by increasing the activationenergy of combustion.

17. What is the energy of a photon of light emitted by abarium chloride emitter?

A. 1.2 × 10–19 J

B. 3.8 × 10–19 J

C. 3.5 × 10–40 J

D. 4.2 × 10–40 J

propellant

stars andbursting charge

stars and

bursting charge

delay fuse

fuse

cardboard barrier

7

GO ON TO THE NEXT PAGE.

03 MCAT FL Test3 06/27/2003 10:25 AM Page 7

Page 8: Full length 3

Passage IV (Questions 18–22)

Musical instruments generate vibrations in the air thatare perceived as musical tones. In many kinds of drums,these vibrations are created by a standing waves in avibrating membrane. In a timpani drum, membrane vibra-tion is coupled to the vibration of an enclosed volume ofair. There may also be a second membrane whose vibrationis coupled to that of the first by the enclosed air space, asin a snare drum. An idealized circular membrane willvibrate at normal mode frequencies given by Equation 1where T is the membrane tension, r is the membraneradius, σ is the mass per unit area of the membrane, and frel

is the relative frequency shown under each mode in Figure1. The pitch of drums can be tuned by adjusting the mem-brane tension.

fmn = �2.40

2π×r

frel� ��σT

��Equation 1

The modes are designated by two numbers, m and n. mindicates the number of diameter nodes, and n indicates thenumber of circular nodes. Several modes of vibration areshown in Figure 1.

Figure 1

18. If the tension of a drum membrane is increased by afactor of four and the radius is increased by a factorof two, then the (1,1) modal frequency would:

A. not change.

B. increase by a factor of 1.59.

C. increase by a factor of 2.

D. increase by a factor of 4.

19. Chladni patterns are formed when fine sand is placedon a vibrating membrane. The regions of localizedsand indicate the:

A. circular nodes only.

B. antinodes.

C. circular nodes and diameter nodes.

D. region equidistant between nodes and antinodes.

20. The speed of sound is inversely proportional to thesquare root of the density of the gaseous mediumthrough which it travels. Which of the followingstatements is true regarding an ideal membraneplayed in a low pressure environment?

A. The sound intensity will decrease because thevibrations will be more rapidly dispersed.

B. The vibration of the membrane will occur withgreater amplitude at the higher modes.

C. The pitch will be higher because the frequencyof vibration will increase.

D. The pitch will remain unchanged because thefrequency of vibration is not changed.

21. The sound waves generated by a drum are:

A. longitudinal waves because the displacement isperpendicular to the direction of propagation.

B. longitudinal waves because the displacement isparallel to the direction of propagation.

C. transverse waves because the displacement isperpendicular to the direction of propagation.

D. transverse waves because the displacement isparallel to the direction of propagation.

22. Which of the following combinations of modal fre-quencies would generate the highest beat frequency?

A. (0,1) and (2,1)

B. (0,1) and (0,2)

C. (1,1) and (3,1)

D. (2,1) and (1,2)

(0,1)1.00

(1,1)1.59

(2,1)2.14

(0,2)2.30

(3,1)2.65

(1,2)2.92

8

GO ON TO THE NEXT PAGE.

03 MCAT FL Test3 06/27/2003 10:25 AM Page 8

Page 9: Full length 3

23. A 2-stage rocket is launched vertically upwards atvelocity v. At height h it discards its first stage in fourfragments and travels faster without changing direc-tion. Which of the following views from above couldbe the velocity vectors of the first stage fragments?

A. C.

B. D.

24. A clown stands with her toes touching a fun house mir-ror with a convex bottom and a concave top. Which ofthe following best describes the distortion of theclown’s image as she walks away from the mirror?

A. Her head will shrink; her feet will grow, thenshrink.

B. Her head will shrink then grow; her feet willshrink, then grow.

C. Her head will grow; her feet will grow.

D. Her head will grow, then shrink; her feet willshrink.

25. The kinetic molecular theory describes gases as com-posed of randomly moving particles. According tothis theory, at constant temperature, the pressure of agas in a cylinder increases when the volume isdecreased because:

A. the kinetic energy of the gas increases.

B. the collisions of gas particles with the cylinderbecome more energetic.

C. the mass of the particles increases to compen-sate for the decrease in volume.

D. the collisions of gas particles with the cylinderincrease in frequency.

26. Which of the following molecules has the same geo-metric configuration of electron pairs about the cen-tral nucleus as CH4?

A. H2O

B. H2CO

C. BH3D. CO2

27. Which of the following solutions has the lowestmolarity?

A. 2N H3PO4B. 2N HCl

C. 1M H3PO4D. 2N H2CO3

••

•••

• ••

• ••

•• •

• •

Questions 23 through 27 are NOT based ona descriptive passage.

9

GO ON TO THE NEXT PAGE.

03 MCAT FL Test3 06/27/2003 10:25 AM Page 9

Page 10: Full length 3

Passage V (Questions 28–31)

Ink jet printers produce high resolution output, at alower cost than laser printers, by generating charged inkdroplets which are then deflected onto a sheet of paper byan electric field. Each droplet deflected by the field strikesthe paper and forms a tiny dot of ink. While a typicalprinted letter requires about 100 drops, an ink jet printer isable to produce drops at a rate of 100,000 per second.

The essential elements of the ink jet printer head areshown in Figure 1. The drop generator produces the inkdroplets, each with a mass of approximately 1.2 × 10–10 kgand a diameter of approximately 30 µm. The drops thenenter a highly precise charging unit which controls thecharge q on each droplet to within 2%, with typical chargesfor drops generated by various ink jet printers rangingfrom –1.0 × 10–13 C to –3.0 × 10–14 C. The chargeddroplets are subsequently passed through the deflectingplates between which a variable electric field is generated.The electronically controlled electric field between theplates is typically varied over a range from 1.0 × 106 N/Cto 5.4 × 106 N/C, and is used to aim the ink droplet at the

paper. (Note: B = �2

µπ0i

r�, F = Eq, and k = 8.99 × 109

N•m2/C2).

Figure 1

28. In which direction would the deflector in Figure 1deflect the ink drop?

A. Upwards in the plane of the page

B. Downwards in the plane of the page

C. Into the plane of the page

D. Out of the plane of the page

29. An ink jet printer deflects a particular ink droplet by1.5 mm in the region of the deflector. Which of thefollowing is a possible value of the work done on thedroplet?

A. No work

B. 4.5 × 10–13 J

C. 4.3 × 10–10 J

D. 8.1 × 10–8 J

30. Suppose that an ink-jet printer head is programmedto produce the letter shown below (note that crosshairs are not produced by the ink jet printer head).

An uniform external electric field pointing upwardsis superimposed on the printer head, and the letter isprinted again. Which of the following diagrams bestrepresents the new printer output?

A. C.

B. D.

31. In order to observe the velocity of the ink droplet, amagnetic field is generated to exactly counter theforce applied to the ink droplet by the electric field.This magnetic field is oriented:

A. towards the left.

B. towards the right.

C. out of the page.

D. into the page.

E

dropgenerator

chargingunit

deflector paper

10

GO ON TO THE NEXT PAGE.

03 MCAT FL Test3 06/27/2003 10:25 AM Page 10

Page 11: Full length 3

Passage VI (Questions 32–37)

The Activity Series is an experimentally derived relativeranking of the elements in order of their reactivities towardsingle displacement reactions. Historically, elements werecategorized into four groups based on their activities relativeto water and aqueous mineral acids as follows:

1. The most active group consisted of those elementswhich displace hydrogen from cold, neutral water (Reac-tion 1).

2. The second most active group was composed ofthose elements which are capable of displacing hydrogenfrom steam but not from liquid water (Reaction 2).

3. The third group’s members do not react with steam,but do displace hydrogen from aqueous acid (Reaction 3).

4. The least active group consisted of those elementswhich do not displace hydrogen from hot, aqueous acidsolution.

A partial list of the elements in each group is shown inTable 1.

M(s) + nH2O(l) → M(OH)n + �n2

� H2(g)

Reaction 1

M(s) + nH2O(g) → M(OH)n + �n2

� H2(g)

Reaction 2

M(s) + nH+(aq) → Mn+ + �n2

� H2(g)

Reaction 3

Group Displaces H+ from: Elements

1 cold water Li, Na, K, Ca

2 steam Mg, Al, Fe, Zn

3 acidic solution Ni, Cd, Sn

4 none Cu, Ag, Au, Pt

Table 1

Later refinements in the activity series were achievedby conducting single displacement reactions of elementswithin a group to determine their relative reactivities.These reactions are designed to measure the propensity ofan element to displace other elements in its group from

their neutral salts (Reaction 4). For example, metallic cop-per was found to displace silver from aqueous silver nitratesolution, while metallic silver remained unreacted in aque-ous copper (II) nitrate solution of similar molarity. Thus,copper was ranked higher than silver in the activity series.Final refinements were accomplished via electrochemicalmeasurements, resulting in the current version of the activ-ity series. After many displacement and electrochemicalexperiments, a complete order of all the stable elementswas determined. An abbreviated activity series is presentedin Table 2.

M(s) + M'Xn(aq) → M'(s) + MXn(aq)

Reaction 4

Activity Element StandardReduction Potential

Most K –2.93

Ca –2.87

Na –2.71

Mg –2.37

Al –1.66

Zn –0.76

Fe –0.44

Cd –0.40

Ni –0.25

Sn –0.14

Cu –0.34

Ag –0.80

Hg –0.86

Pt –1.20

Least Au –1.50

Table 2

32. In which of the following ways do the group 1 ele-ments in Table 1 differ from all of the elements in theother groups?

A. They are stronger Lewis acids.

B. They are stronger reducing agents.

C. They are stronger oxidizing agents.

D. They are stronger Lewis bases.

11

GO ON TO THE NEXT PAGE.

03 MCAT FL Test3 06/27/2003 10:25 AM Page 11

Page 12: Full length 3

33. Which one of the following is a valid representationof a single displacement reaction?

A. 4Fe(s) + 3O2(g) → 2Fe2O3(s)

B. Cu(OH)2(s) + 2HCl(aq) →CuCl2(aq) + 2H2O(l)

C. Cu(OH)2(s) → CuO(s) + H2O(g)

D. Cu(OH)2(s) + Fe(s) → Cu(s) + Fe(OH)2(s)

34. Which of the following statements is true of an elec-trochemical cell based on the reaction below?

3Zn(s) + 2Al(OH)3(s) → 3Zn(OH)2(s) + 2Al(s)

A. The cell is galvanic with a cell potential of–0.90V.

B. The cell is galvanic with a cell potential of+0.90V.

C. The cell is electrolytic with a cell potential of–0.90V.

D. The cell is electrolytic with a cell potential of+0.90V.

35. The value of the variable n in Reaction 1 equals:

A. the oxidation state of the metal in the product.

B. the oxidation state of the nonmetal in the prod-uct.

C. the oxidation state of the metal in the reactant.

D. the oxidation state of the nonmetal in the reac-tant.

36. A researcher attempts to determine the relative activ-ity of the nonmetallic elements. Which of the follow-ing reactions, if spontaneous, would confirm thatchlorine is more active than bromine?

A. HBr(aq) + KCl(s) → KBr(aq) + HCl(aq)

B. AgCl(aq) + HBr(aq) → AgBr(s) + HCl(aq)

C. 2HBr(aq) + Cl2(aq) → 2HCl(aq) + Br2(aq)

D. Br2(l) + Cl2(g) → 2BrCl(l)

37. The activity of the alkali metals increases withatomic number. This trend in activity is most directlya result of a parallel trend in:

A. effective nuclear charge.

B. atomic radius

C. atomic mass

D. neutron/proton ratio

12

GO ON TO THE NEXT PAGE.

03 MCAT FL Test3 06/27/2003 10:25 AM Page 12

Page 13: Full length 3

Passage VII (Questions 38–44)

In 1965, Boris Deryagin reported the discovery of anunusual substance formed during the condensation ofwater vapor in quartz capillaries. The material, called poly-water, appeared to be a polymer of water monomers anddiffered from normal water in a number of ways. It had afreezing point of –40° C and solidified into a glass-likesolid with substantially less volumetric expansion than thatof ordinary water upon freezing. It had a density 40%greater than water and a refractive index of 1.48.

An intricate apparatus was used to produce the poly-water. Ordinary distilled water was placed in a chamberheld at 160° C with pressure below atmospheric pressure.This chamber was connected to a second chamber by atube held at 500° C in order to prevent the passage of liq-uid water. The second chamber was held at 0° C and con-tained a drawn quartz capillary in which the water vaporcondensed, forming polywater.

Hypothesis 1

Deryagin proposed that polywater was a polymer ofwater monomers arranged in a network of hexagonal units.The polymerization was catalyzed by the silicate surfaceof the quartz capillary.

Hypothesis 2

Another researcher was skeptical. Analysis indicatedthat polywater was merely a solution of water and dis-solved particles including silicon, carbon dioxide, and sub-stantial concentrations of ions (Na+ and Cl–). Thesecontaminants dissolved from the quartz capillary and frommaterials used in the apparatus.

(constants for normal water : density = 1 g/cm2, indexof refraction = 1.33 , freezing point depression constant Kf= 1.86°Cm–1)

38. Hypothesis 1 proposes that polywater is a repeatingstructure of H2O monomers. What is the formalcharge on hydrogen in this structure?

A. –1

B. +1

C. –2

D. +2

O

H

OH

OH

O

H

OH

OH

HO

HO

H

OH

OH

O

H

HO

HO

H

Proposed Structure of Polywater

-2

distilledwater

< 1 Atm160°C

quartzcapillary

0°C

500°C

13

GO ON TO THE NEXT PAGE.

03 MCAT FL Test3 06/27/2003 10:25 AM Page 13

Page 14: Full length 3

39. According to the passage, polywater has all of thefollowing properties EXCEPT:

A. light entering polywater from air will bend moretowards the normal than light entering normalwater.

B. polywater can be produced in a variety of con-ditions.

C. 1 µg of water has a larger volume than 1 µg ofpolywater, at room temperature.

D. polywater exists in the liquid state at 0° C.

40. Which of the following changes in the experimentalapparatus would increase the rate of water vapor pro-duction in the first chamber of the experimentalapparatus? (All other conditions kept constant.)

A. Decreasing the temperature

B. Decreasing the pressure

C. Decreasing the cross-sectional area of the tubeconnecting the two chambers

D. Adding NaCl to the distilled water in the firstchamber

41. Assume that Hypothesis 2 is correct. Compared tonormal water, polywater would have a:

A. lower vapor pressure and higher boiling point.

B. lower vapor pressure and lower boiling point.

C. higher vapor pressure and lower boiling point.

D. similar vapor pressure and boiling point.

42. The pressure (Pi) in the first chamber is decreased byraising a piston and increasing the volume of thechamber from Vi to Vf. If the temperature is kept at160 °C, which of the following values solves for thenew pressure (Pf) in the chamber?

A. �P

ViV

f

i�

B. Pi – �P

ViV

f

i�

C. �nVR

i

T�

D. ViTi – VfTf

43. Which of the following would be most likely to occurif the second chamber was kept at 50° C instead of0° C?

A. The water vapor would not condense.

B. Liquid water would be able to travel through thetube between chambers.

C. A different polymer of polywater would form.

D. The water vapor would condense more slowly.

44. Which of the following pieces of evidence wouldmost support Hypothesis 1?

A. The mass of the quartz capillary did not changethroughout the experiment.

B. Filtration of the polywater increased its freezingtemperature.

C. The polywater was found to differ from normalwater in its boiling point.

D. The second chamber could be kept at 50° C withsimilar results.

14

GO ON TO THE NEXT PAGE.

03 MCAT FL Test3 06/27/2003 10:25 AM Page 14

Page 15: Full length 3

Passage VIII (Questions 45-49)

The nuclei of certain unstable isotopes will sponta-neously decay, producing a more stable nucleus and releas-ing a particle or quantity of energy. Alpha decay releases ahelium nucleus, beta decay emits an electron, whilegamma decay is the emission of a high energy photon.Each type of radioactive decay is characterized, in part, bythe half-life of the radioactive material—the time requiredfor half of the nuclei in a sample to undergo decay. Exam-ples of such decays are shown in Figure 1.

alpha decay U → Th + α

beta decay Co → Th + β

gamma decay Ni* → Ni + γ

Figure 1

A Geiger counter can be used to detect the decay ofradioactive materials. A simple Geiger counter consists ofa hollow metal cylinder with a wire along its axis. Thecylinder is filled with low pressure argon gas and a highvoltage difference is applied between the wire and thecylinder. When alpha, beta, or gamma radiation passesthrough the cylinder, it interacts with the gas particles andleads to the formation of ions which cause a dischargebetween the wire and the cylinder. The consequent currentmay be used to drive a speaker, producing the characteris-tic clicking sound of the Geiger counter each time a pulseof current occurs. The Geiger counter circuitry is shown inFigure 2.

Figure 2

45. Beta radiation differs from gamma rays in the fol-lowing way:

A. gamma rays can penetrate the Geiger countercylinder, while beta rays cannot.

B. beta particles and gamma rays will be deflectedin opposite directions by a magnetic field.

C. beta particles and gamma rays will be deflectedin opposite directions by an electric field.

D. beta particles will be deflected in a magneticfield while gamma rays will not.

46. The half-life of 32P is 14.28 days. What fraction of aninitially pure sample will have decayed after 28.56days?

A. 1/4

B. 1/2

C. 3/4

D. 1

47. Assume that the values for V and R are known. Whatother information is needed to calculate the voltagedrop across the speaker?

A. The resistance of the speaker’s internal cir-cuitry.

B. The duration of the discharge between the wireand the cylinder.

C. The current flowing through the resistor.

D. No other information is required.

metal cylinder

wire

speaker

V

R

64

28

64

28

0

–1

60

28

60

27

4

2

234

90

238

92

15

GO ON TO THE NEXT PAGE.

03 MCAT FL Test3 06/27/2003 10:26 AM Page 15

Page 16: Full length 3

48. Which of the following statements is true concerningtwo isolated radioactive samples examined sepa-rately with a Geiger counter?

A. The sample with the shorter half-life will pro-duce a higher frequency of clicks.

B. The sample with the longer half-life will pro-duce a higher frequency of clicks.

C. The sample with the shorter half-life will gener-ate a larger current.

D. The half-life cannot be determined from theclick frequency alone.

49. A Geiger counter is best suited for which of the fol-lowing applications:

A. comparing the relative magnitudes of radioac-tivities of two nuclear waste depositories.

B. spatially locating a radioactive isotope injectedinto a patient.

C. calculating the total energy of a radioactive par-ticle.

D. determining the identity of various types ofradioactivity.

16

GO ON TO THE NEXT PAGE.

03 MCAT FL Test3 06/27/2003 10:26 AM Page 16

Page 17: Full length 3

50. According to the reaction coordinate diagram shownbelow, which of the following forward reactionswould proceed most rapidly at high temperatures?

A. A

B. B

C. C

D. D

51. 1M NaOH is added to a solution containing 1M Ag+,1M Al3+, 1M Mg2+, and 1M Mn2+. Given the solu-bility data shown below, which of the following willprecipitate first?

Ksp

AgOH 1.5 × 10–8

Al(OH)3 3.7 × 10–15

Mg(OH)2 1.2 × 10–11

Mn(OH)2 2.0 × 10–13

A. AgOH

B. Al(OH)3C. Mg(OH)2D. Mn(OH)2

52. An object at rest with the shape of an equilateral tri-angle is subject to three external forces as shown inthe diagram. If the object does not exhibit transla-tional motion, what must F and θ be, respectively?

A. 20 N, 120°

B. 20 N, 165°

C. 10 N, 120°

D. 10 N, 165°

53. A solar-power collector has an area of 10 cm2 facingthe sun. If the intensity of the sunlight incident uponthis surface is 1.5 kW/m2, what is the maximumenergy the device can supply in one hour?

A. 1500 J

B. 15000 J

C. 5400 J

D. 54000 J

54. For the chemical reaction A + B → C, the initialrates of reaction under different conditions are givenbelow. What is the overall order of this reaction?

initial [A] initial [B] initial rate of(mol/L) (mol/L) reaction (mol/L•min)

0.50 0.10 0.01

1.00 0.40 0.16

1.00 1.60 0.64

A. 1

B. 2

C. 3

D. 4

10 N

30 N

F

θ75°

285°

A

B

C

D

Ene

rgy

Reaction Coordinate

Questions 50 through 54 are NOT based ona descriptive passage.

17

GO ON TO THE NEXT PAGE.

03 MCAT FL Test3 06/27/2003 10:26 AM Page 17

Page 18: Full length 3

Passage IX (Questions 55–59)

A hovercraft is a versatile vehicle capable of travelingover land, water, or any other essentially flat surface. Thehovercraft consists of a body or hull onto which a rotor (liftfan) is mounted. The lift fan provides the vertical lift bypropelling air into an area beneath the hovercraft called theskirt. The pocket of air in the skirt supports the movinghovercraft and reduces the friction between the vehicle andthe ground to almost zero. As such, there is no contactbetween the hovercraft and the ground.

A second fan, which generates a horizontal thrust, pro-pels the hovercraft forward. Rudders which direct the airflowfrom this second fan are used by the pilot to control themovement of the hovercraft. The horizontal movement of thehovercraft is opposed by air resistance which generates aero-dynamic drag. (1 atm = 1.01 × 105 Pa = 1.01 × 105 N/m2)

55. A 600 kg hovercraft hovers 0.6 meters above theground. The rectangular hovercraft is 4 meters longand 2 meters wide with a skirt which hangs from theedge to a distance of 4 cm above the ground. What isthe average pressure of air in the skirt?

A. 735 Pa

B. 980 Pa

C. 47.040 kPa

D. 101.735 kPa

56. The horizontal thrust fan of a hovercraft travelingacross a frozen lake in a large circle suddenly fails.Which of the following best describes the path of thehovercraft (as one looks down from above) immedi-ately following thrust fan shutoff?

A. C.

B. D.

57. A 600 kg hovercraft traveling across a frozen lakeaccelerates from rest to a speed of 60 kilometers perhour in 4 seconds. What is the average force pro-duced by the thrust fan?

A. 4.2 N

B. 145 N

C. 2500 N

D. 9000 N

58. The thrust fan of a hovercraft is turned on at time t1and turned off at time t2. In the absence of air resis-tance, which of the following graphs represents thevelocity of this hovercraft as a function of time?

A. C.

B. D.

59. Which of the following conditions must be true for thethrust fan to propel the hovercraft horizontally?(Assume that the air is not compressed in the process.)

A. The air intake velocity must be lower than theair output velocity.

B. The air intake velocity must be greater than theair output velocity.

C. The air intake volume must be lower than the airoutput volume.

D. The air intake pressure must be greater than theair output pressure.

t1 t2ve

loci

tyt1 t2

velo

city

t1 t2

velo

city

t1 t2ve

loci

ty

18

GO ON TO THE NEXT PAGE.

03 MCAT FL Test3 06/27/2003 10:26 AM Page 18

Page 19: Full length 3

Passage X (Questions 60–65)

The Earth’s atmosphere reaches hundreds of kilome-ters above the surface of the planet. The lowest layer, thetroposphere, extends from the ground to a height ofapproximately 12 km. Air pressure within the tropospheredecreases with height above the ground, accompanied by aparallel trend in air density. The decrease in density hasimportant consequences for the dissipation of air pollutionfrom industrial smoke stacks. The gas from the stack istypically hotter and less dense than the surrounding air andrises. As a parcel of hot air rises, it expands approximatelyadiabatically doing work on the surrounding air. Thisresults in a decrease in both its temperature and its density.

Figure 1

A smoke stack functions to expel gaseous waste prod-ucts from a chemical process. It is also an important meansof removing heat from a reaction mixture. The heat corre-sponding to a change in temperature of a gas at constantpressure is given by Q=nCp∆T, where Q is the heat addedto the gas, n is the number of moles of gas, Cp is the molarheat capacity of a particular gas at constant pressure, and∆T is the change in temperature. At atmospheric pressure,the molar heat capacity for steam, H2O (g) is approxi-mately four times that of air.

60. The decrease in temperature of a parcel of hot air ris-ing from a smoke stack is a result of:

A. conductive heat losses to the surrounding coolerair.

B. convective heat losses to the surrounding coolerair.

C. an increase in kinetic energy.

D. the expansion of the parcel.

61. Ozone molecules in the stratosphere absorb ultravio-let radiation and photodissociate into an oxygenmolecule (O2), and a free oxygen atom (O). Which ofthe following best describes this phenomenon?

A. The wavelength of the ultraviolet radiation isgreater than the cutoff wavelength needed forphotodissociation.

B. The frequency of the ultraviolet radiation isgreater than the cutoff frequency needed forphotodissociation.

C. The frequency of the ultraviolet radiation is lessthan the cutoff frequency needed for photodis-sociation.

D. The wavelength of the ultraviolet light is lessthan the cutoff frequency.

62. According to Figure 1, what is the approximate rela-tionship between the energy content of a liter of air at12 km and that of a liter of air at 120 km?

A. The energy content in a liter at 12 km is greaterbecause the mass is greater.

B. The energy content in a liter at 120 km is greaterbecause the temperature is higher.

C. The energy content in a liter at 120 km is greaterbecause the air is less dense.

D. The energy contents are equal because the vol-umes are equal.

500

400

300

200

100

0

Tem

pera

ture

(K

)

100 kP

80 kP

60 kP

40 kP

20 kP

0

Pres

sure

(Pa

)

0 20 40 60 80 100 120 140Altitude (km)

pressure

temperature

tropo

sphe

re

strato

sphe

re

mesos

phere

therm

osph

ere

19

GO ON TO THE NEXT PAGE.

03 MCAT FL Test3 06/27/2003 10:26 AM Page 19

Page 20: Full length 3

63. Two identical balloons of negligible mass are teth-ered at altitudes of 2000 meters and 2600 meters,respectively. The balloons are filled with helium gasto equal volumes. Which of the following statementsis true concerning the buoyant force acting on eachballoon?

A. The buoyant force on the balloon at 2600 meterswill be greater.

B. The buoyant force on the balloon at 2000 meterswill be greater.

C. The buoyant forces on the two balloons will beequal.

D. The relationship between the buoyant forcescannot be determined.

64. A combustion engine in a production plant is sur-rounded by pipes carrying water that function to coolthe engine. The water is converted to steam and flowsthrough a long vertical pipe to be released into theatmosphere. Heat is transferred from the engine tothe atmosphere by the following means:

A. conduction then convection

B. convection then conduction

C. radiation then convection then conduction

D. conduction then convection then conduction

65. At constant pressure, the ratio between the volumeexpansion of a mole of gas with a high Cp to that ofa mole of gas with a low Cp for a given heat inputwill be:

A. greater than 1.

B. less than 1.

C. equal to 1.

D. cannot be determined.

20

GO ON TO THE NEXT PAGE.

03 MCAT FL Test3 06/27/2003 10:26 AM Page 20

Page 21: Full length 3

Passage XI (Questions 66–72)

A student conducts a chemical analysis of the compo-nents of a popular soft drink. The beverage label showsthat the drink contains carbonated water, phosphoric acid,caffeine, and caramel color, but does not indicate the con-centrations of these chemicals.

Carbonic Acid Phosphoric Acid

MW 62.03 98.00

mp (°C) n/a 42.35

Ka (1) 4.3 × 10–7 (1) 7.52 × 10–4

(2) 5.61 × 10–11 (2) 6.23 × 10–8

(3) 2.2 × 10–13

Formula H2CO3 H3PO4

Table 1

Dissolved carbon dioxide will react reversibly withwater to form carbonic acid. In an attempt to analyze thebeverage composition, the student conducts the followingexperiments on a one liter sample of the beverage.

Experiment 1

The sample is placed in a sealed beaker cooled to 10°C and a vacuum is created in the space above the beverage.The gas pumped from this space is passed through a solu-tion of BaCl2, producing a white precipitate. The processcontinues until no more precipitate forms. The precipitateis dried and found to have a mass of 9.5 grams.

Experiment 2

The remaining solution left in the sealed beaker is thentitrated with 0.01 M NaOH to give the titration curveshown in Figure 1.

Figure 1

66. What is the approximate concentration of phosphoricacid in a 0.5 L sample of the beverage?

A. 0.005 M

B. 0.025 M

C. 0.5 M

D. 2.5 M

67. The student uses the data from Experiment 1 andExperiment 2 to calculate the initial pH of the bever-age. If a significant quantity of precipitate was lost inthe drying process of Experiment 1, the calculatedpH:

A. would be less than the actual pH.

B. would be greater than the actual pH.

C. would be the same as the actual pH.

D. would differ from the actual pH in a randommanner.

12

11

10

9

8

7

6

5

4

3

2

1

00 500 750 1000250

pH

0.01 M NaOH (mL)

A

B

C

D

21

GO ON TO THE NEXT PAGE.

03 MCAT FL Test3 06/27/2003 10:26 AM Page 21

Page 22: Full length 3

68. Which of the following plots best indicates the com-position of phosphoric acid in the 1 Liter sample dur-ing the titration?

A.

B.

C.

D.

69. Ba3(PO4)2 is completely insoluble in aqueous solu-tion. Addition of barium chloride to a phosphoricacid solution will decrease the pH because:

A. barium acts as a Lewis base.

B. barium chloride reduces the concentration ofhydrogen ions in solution.

C. chloride ion acts to stabilize the hydrogen ionsin solution.

D. the precipitation of barium phosphate drives theacid dissociation to completion.

70. Which region of the graph in Figure 1 provides thebest buffering around neutral pH?

A. A

B. B

C. C

D. D

71. Why did the student choose to keep the sealed beakerof beverage at 10° C while vacuuming the CO2 fromsolution?

A. To reduce dipole-dipole interactions betweenthe water and the CO2.

B. To ensure that CO2 did not expand explosively.

C. To minimize the loss of water vapor from thesolution.

D. To increase the kinetic energy of CO2 and makeits removal more efficient.

72. In the body, rapid breathing (hyperventilation) leadsto a decrease in the concentration of carbon dioxidein the blood. This will tend to:

A. decrease the pH of the blood.

B. increase the pH of the blood.

C. increase the concentration of HCO3– in the

blood.

D. increase the concentration of O2 in the blood.

Frac

tion

1.0

0.8

0.6

0.4

0.2

0.00 1 2 3 4 5 6 7 8 9 10 11 12

H3PO4

H2PO4-

HPO42-

pH

Frac

tion

1.0

0.8

0.6

0.4

0.2

0.00 1 2 3 4 5 6 7 8 9 10 11 12

H3PO4 H2PO4-

HPO42-

pH

pH

Frac

tion

1.0

0.8

0.6

0.4

0.2

0.00 1 2 3 4 5 6 7 8 9 10 11 12

H3PO4 H2PO4-

HPO42-

Frac

tion

1.0

0.8

0.6

0.4

0.2

0.00 1 2 3 4 5 6 7 8 9 10 11 12

HPO42- H2PO4

-

H3PO4

pH

22

GO ON TO THE NEXT PAGE.

03 MCAT FL Test3 06/27/2003 10:26 AM Page 22

Page 23: Full length 3

73. Given the following ∆H° values, calculate the ∆H° offormation of Fe2O3 (s).

6Fe2O3(s) → 4Fe3O4(s) + O2(g) +472.0 kJ mol–1

3Fe(s) + 2O2(g) → Fe3O4(s) –1118.4 kJ mol–1

A. +667 kJ mol–1

B. –1559 kJ mol–1

C. –824 kJ mol–1

D. –667 kJ mol–1

74. A 5-kg mass M is being raised from the ground to thetop of the inclined plane using the set-up shown inthe diagram below. Assuming that the inclined planeis frictionless, what is the work done by the force F?

A. 490 J

B. 490�3� J

C. 490�2� J

D. J

75. In the circuit below, R1 = R2 = 10 ohms. RemovingR1 will have which of the following effects on thecurrent through R2?

A. Decrease to 1/2

B. No change

C. Double

D. Increase by factor of 10

76. An automobile reaches a velocity of 30 m/s afteraccelerating at 4 ms–2 for 5 seconds. What was itsinitial velocity?

A. –10 m/s

B. 0 m/s

C. 10 m/s

D. 20 m/s

77. A gibbon (lesser ape) of mass m and arm length lreaches to a branch level with its shoulder and startsto swing with its arm fully extended. At the bottom ofthe swing, its velocity is:

A. �2�g�l�

B. �12

�mv2

C. �ml

2�

D. 2π��gl��

STOP. IF YOU FINISH BEFORE TIME IS CALLED,CHECK YOUR WORK.YOU MAY GO BACK TO ANYQUESTION IN THIS SECTION ONLY.

V

R1 R2

490��2�

F

M

10 m

45° 30°

Questions 73 through 77 are NOT based ona descriptive passage.

23

03 MCAT FL Test3 06/27/2003 10:26 AM Page 23

Page 24: Full length 3

03 MCAT FL Test3 06/27/2003 10:26 AM Page 24

Page 25: Full length 3

Verbal ReasoningTime: 85 MinutesQuestions 78–137

DO NOT BEGIN THIS SECTION UNTIL YOU ARE TOLD TO DO SO.

03 MCAT FL Test3 06/27/2003 10:26 AM Page 25

Page 26: Full length 3

Passage I (Questions 78–83)

The anthropomorphic bias of those who would relegatemarsupials to an inferior evolutionary status is most appar-ent in their recourse to data on brain structure and behav-ior. Unlike humans and other placentals, marsupials lackthe corpus callosum, which facilitates inter-hemispheretransfer of data acquired through the senses. Yet it cannotbe inferred that marsupials are thus deprived of such func-tion. Didelphis Virginiana, one of the opossums, makesuse of the anterior commissure, an adaptation that is alsofound in reptiles and monotremes. Diprodontons, includ-ing kangaroos and koalas, supplement the anterior com-missure with the fasciculus aberrans. While the modes ofneocortical interconnection may be diverse, the work ofJohnson, Heath and Jones points to the conclusion that,functionally speaking the cortices and neocortices of bothgroups of mammals exhibit parallel connections. Parkeralso notes “a similar range of brain size to body weightratios and of neocortical expansion.”

Another stigma borne by marsupials is the consensusthat they are less intelligent than placentals. Yet Williamsargues that, all else being equal, natural selection willfavor instinctive over learned behavior as being more bio-logically efficient and that it is the accidental death of theyoung that is the prime selective pressure for the evolutionof intelligence. Seen in this light, marsupials have a com-petitive edge; their gestation period is brief and the youngremain in the pouch for an extended period exposed onlyto those dangers which also affect the mother. There theyare directly exposed to the mother’s food supply and canobserve her behavior at leisure. Placentals, on the otherhand, not only have a longer gestation period but, oncetheir young are born, must often leave while foraging.Such absences increase the risk of mortality and decreasethe opportunity to learn. Thus, among placentals, selectionwould favor the apparent intelligence in the young andprotective behavior in the mother.

Marsupials are not known to exhibit maternal protec-tive behavior. In fact, Serventy has reported that frightenedfemale kangaroos will drop their pouch-young as they flee,drawing a predator’s attention to the less able offspringwhile the adult escapes. This behavior, whether purposefulor accidental, instantaneously relieves the female marsu-pial of the mechanical difficulties of pregnancy with whichher placental counterpart would be burdened, while mar-supials can replace any lost young quickly. Thus, in theabsence of any need for close maternal supervision, sacri-ficing their offspring in this manner may well have beenfavored in selection. Pointing to the absence of the “virtue”of maternal protectiveness in marsupials is an instance ofhow mistaken are those theorists who see similarities withhumans as marks of evolutionary sophistication.

78. The author’s primary purpose in this passage is to:

A. correct some common misconceptions aboutmarsupials.

B. argue against the view that marsupials represent aless developed evolutionary stage than placentals.

C. provide support for the proposition that marsu-pials have adapted more successfully to theenvironment.

D. determine the place of marsupials in the evolu-tionary hierarchy.

26

GO ON TO THE NEXT PAGE.

5

10

15

20

25

30

35

40

45

50

VERBAL REASONING

DIRECTIONS: There are nine passages in this VerbalReasoning Section. Each passage is followed by sev-eral questions. After reading a passage, select the onebest answer to each question. If you are not certain ofan answer, eliminate the alternatives that you know tobe incorrect and then select an answer from theremaining alternatives. Indicate your selection byblackening the corresponding oval on your answerdocument.

03 MCAT FL Test3 06/27/2003 10:26 AM Page 26

Page 27: Full length 3

79. According to the passage, which of the followingfavor(s) the development of intelligence as a trait ofplacental mammals?

I. The need to leave their young while foraging

II. The comparatively great risk of accidentaldeath of the young

III. The opportunity for the young to observethe mother at leisure

A. I only

B. III only

C. I and II only

D. II and III only

80. The author’s attitude toward those who consider mar-supials to occupy an inferior evolutionary positionwould most probably be one of:

A. criticism because they ignore evidence that mar-supials are more intelligent than usually sup-posed.

B. disagreement because current studies supportthe opposite view.

C. disagreement because they apply human stan-dards in an inappropriate context.

D. agreement, but on the basis of marsupials’ lackof maternal protective behavior rather than theirbrain structure.

81. The author uses the word “virtue” (line 48) in order to:

A. remind the reader that the word has specializedconnotation in this context.

B. ridicule the thinkers whom she is attacking inher conclusion.

C. introduce the idea that moral concepts can beapplied only to humans.

D. indicate that the term reflects a mode of think-ing with which she disagrees.

82. With which of the following statements would theauthor be most likely to agree?

A. Maternal protectiveness is a trait common to allhigher mammals.

B. Any physical or behavioral trait in animalsshould be evaluated primarily in terms of itscontribution to species survival.

C. Current conceptions of evolution must be mod-ified to account for new data.

D. Evolution is a progressive process culminatingin the dominance of learned over instinctivebehavior.

83. According to the passage, similarities between mar-supials and placentals will most likely be found in:

A. brain function.

B. brain anatomy.

C. maternal behavior.

D. selection for intelligence.

27

GO ON TO THE NEXT PAGE.

03 MCAT FL Test3 06/27/2003 10:26 AM Page 27

Page 28: Full length 3

Passage II (Questions 84–90)

By now the image of California in decline looms aslarge in the conventional media wisdom as the GoldenState—triumphant clichés of a generation ago— “this ElDorado,” as Time magazine had put it in 1969, that was tobe “the mirror of America as it will become.” Hardly any-one mentions the sunshine these days, or the beaches, or thebeautiful young families around the pool, or the newlifestyles that all Americans will soon emulate, or how theUniversity of California is wall-to-wall with cyclotrons andNobel laureates, or how the state’s higher-education systemis accommodating absolutely all comers at little or no cost.

Today, California classrooms are among the mostcrowded in the country; many schools operate withoutlibraries, without counselors, without nurses, without art ormusic, with greatly diminished curricular offerings. Andwhat’s true for the schools is true for the other services thathave no powerful constituencies: children’s protective ser-vices, probation, public health. Many cities have shut downswimming and wading pools because they cannot be safelymaintained, and fenced playgrounds have been shut becauseof the danger presented by cracked and splintered structures.

The list could be extended indefinitely. As thousandsof professors receive golden handshakes from the Univer-sity of California and California State University, amongthem some of the stars recruited in the go-go Fifties, thecrowding in the lecture halls has increased and the lines atthe classroom door have gotten longer and longer (“Don’tpanic,” says the T-shirt on a student waiting to enroll at aSacramento junior college, but many have been in linesince four in the morning). U.C. tuition, which wasroughly $800 a year in the early 1980s, is now over $4,000,a figure not out of line with tuitions at public colleges inother states but a far cry from the cost of a California stateeducation in the golden days—and it is almost certain toincrease again next year. More than 200,000 students—roughly 10 percent—have vanished from the rolls of thestate’s colleges and universities in the past two years.

While per capita tax revenues have been effectivelyfrozen, and while they have declined relative to other states,client rolls for state services—schools, prisons, Medicaid,welfare—have been rising faster than population, leaving astructural gap that no one has yet confronted, much lessclosed. Again this year, the governor and legislature bor-rowed $7 billion from the banks and rolled over a $5 billionbudget deficit, for which few politicians have proposed anyremedies. Thanks to the deficit, California, which a decadeago, had one of the highest bond ratings in the country, hasone of the lowest. “Were California a corporation,” saidJohn Vasconcellos, the chairman of the State Assembly

Ways and Means Committee, “it would have little option butto initiate some sort of bankruptcy proceeding.”

The new image of California is familiar enough: a statesuffering from earthquakes, fires, drought, floods, urbanriots, dirty air, schools as overcrowded as the freeways; alegislature—once said to be the nation’s most professionaland progressive—oozing with corruption and stuck in thebudgetary gridlock; and of course, recession, unemploy-ment, chronic budget deficits, and financial calamity.

For those who know their Nathaniel West, their Ray-mond Chandler, and their Joan Didion, the Californiaapocalypse imagery is hardly new; it was always there onthe dark side of the dream. This was the place, as Didionwrote back in the 1960s, “in which a boom mentality anda sense of Chekhovian loss meet in uneasy suspension; inwhich the mind is troubled by some buried but ineradica-ble suspicion that things better work here, because here,beneath that immense bleached sky, is where we run out ofcontinent.” Los Angeles has burnt before.

If you believe people like Governor Wilson, most of thestate’s problems were created somewhere else, usually inWashington, where the Clinton Administration has, on theone hand, cost California hundreds of thousands of jobsthrough excessive defense cuts and, on the other, allowed ahorde of illegal immigrants to overrun the state’s schoolsand health facilities without paying them for the immensecosts that come with them…much has been changed in Cal-ifornia since the days of West and Chandler, but the capac-ity for denial and self-deception is undiminished.

In fact, California’s trouble is at once more prosaic andmore complex than the political rhetoric claims or theapocalyptic imagery suggests. It began before the recentrecession, the big 1991 fire in the Oakland hills or the SanFrancisco earthquake of 1989 (itself a rerun of a classic),before those L.A. cops beat up Rodney King or the riot andthe fire that followed their acquittal in the first trial, beforethe eight-year drought that still may not be over. And con-trary to what a lot of Californians believe, a lot of the dam-age didn’t just happen to us: we inflicted it on ourselves.

84. The central purpose of this passage is to:

A. ascribe California’s economic and social prob-lems to a set of measurable factors.

B. sketch the myriad difficulties facing Californiatoday.

C. contrast California’s economic and social prob-lems with those of other states.

D. to dispute the contention that California’s prob-lems were caused by the federal government.

28

GO ON TO THE NEXT PAGE.

5

10

15

20

25

30

35

40

45

50

55

60

65

70

75

80

85

03 MCAT FL Test3 06/27/2003 10:26 AM Page 28

Page 29: Full length 3

85. Why does the author mention Governor Wilson’sopinions in paragraph seven?

A. He wishes to underscore the economic plightthat California has been submerged in throughfederal policy.

B. He wants to highlight a point of view withwhich he will disagree.

C. He is portraying California’s plight as a naturalresult of unfortunate circumstances.

D. He wishes to show that California’s governor isas guilty as any other party in the demise of Cal-ifornia’s economy.

86. The strongest contrasts between California’s educa-tional system in the past and that of today can clearlybe seen in:

A. the quality of staff and equipment and the ratioof students to teachers.

B. the availability of higher education to more peo-ple and the atypically high tuition compared tothe rest of the nation.

C. the lack of distinguished professors andincreased tuition costs.

D. the decrease in student enrollment at state uni-versities and the ratio of students to teachers.

87. With which of the following statements concerningCalifornia would the author most likely agree?

A. The state services with weak constituencieshave suffered.

B. The image of California as an alternate heavenand hell is not universal.

C. The state legislature has always been ineffective.

D. The recent waves of immigration are the key tounderstanding the overtaxed state infrastructure.

88. This passage argues that California has amassedoverwhelming fiscal and social problems since the1960s. The author uses which of the following tosupport this assertion?

I. California has repeatedly borrowed fundsto cover its budget shortfalls.

II. Washington has played a major role in thedecline of the California state economy.

III. Californian’s demands for state serviceshave far outstripped the state’s ability toprovide them.

A. I only

B. II only

C. II and III only

D. I and III only

89. Which of the following most weakens the image ofCalifornia as an “El Dorado”?

A. “…the California apocalypse imagery…wasalways there on the dark side of the dream…”

B. “…the University of California is wall-to-wallwith cyclotrons and Nobel laureates…”

C. “…U.C. tuition…is now over $4,000, a figurenot out of line with tuitions at public colleges inother states…”

D. “…the new lifestyles that all Americans willsoon emulate…”

90. It can be inferred from the passage that compared toother states, California:

A. had held one of the highest bond ratings.

B. continues to make public education affordableto its residents.

C. has a progressive and professional legislature.

D. has been successful in balancing budgets.

29

GO ON TO THE NEXT PAGE.

03 MCAT FL Test3 06/27/2003 10:26 AM Page 29

Page 30: Full length 3

Passage III (Questions 91–97)

The mind, just like the body, has its needs. The needsof the body are the foundations of society; those of themind are its amenities. While government and laws pro-vide for the safety and well-being of men when they gathertogether, the sciences and the arts, which are less despoticbut perhaps more powerful, spread garlands of flowersover the iron chains that bind them, stifle in them the sensefor that original liberty for which they seem to have beenborn, cause them to love their own enslavement, and turnthem into so-called “civilized people.” Necessity raisedthrones; the sciences and the arts have strengthened them.O earthly powers: cherish talents and protect those whocultivate them. O civilized people, cultivate them: youhappy slaves owe to them that delicate and refined taste ofwhich you are so proud, that gentleness of character andurbanity of manner which make relations among you soamiable and easy—in other words, that semblance of allthe virtues, none of which you actually possess…

…How pleasant it would be to live among us, if ourexternal appearance were always a reflection of what is inour hearts, if decency were virtue, if our maxims served asour rules, and if true philosophy were inseparable from thetitle of philosopher! But so many qualities are seldomfound together, and virtue hardly ever walks in such greatpomp. Richness of adornment may be the mark of a manof taste, but a healthy, robust man is known by other signs:it is beneath the rustic clothes of a farmer, and not the giltof a courtier, that strength and vigor of the body will befound. Ornamentation is just as foreign to virtue, which isthe strength and vigor of the soul. The good man is an ath-lete who prefers to compete in the nude: he disdains allthose vile ornaments which would hinder the use of hisstrength, ornaments which were for the most part inventedonly to hide some deformity.

Before art had molded our manners and taught our pas-sions to speak an affected language, our customs were rus-tic but natural, and differences in conduct revealed clearlydifferences in character. Human nature, basically, was nobetter, but men found security in being able to see througheach other easily, and this advantage, which we no longerappreciate, spared them many vices.

Now that more subtle refinements and more delicatetaste have reduced the art of pleasing to set rules, a baseand deceptive uniformity prevails in our behavior, and allminds seem to have been cast in the same mold. Inces-santly politeness and propriety make demands on us, andincessantly we follow usage but never our own inclina-tions. We no longer dare to appear as we are, and under thisperpetual constraint, the men who form this herd called

society, when placed in the same circumstances, will all actsimilarly unless stronger motives direct them to do other-wise. Therefore we will never know well those with whomwe deal, for to know our friends we will have to wait forsome crises to arise—which is to say that we will have towait until it is too late, as it is for these very crises that itis essential to know one’s friends well.

What vice would not accompany this uncertainty? Nomore sincere friendships, no more genuine esteem, nomore well-based confidence. Suspicion, offenses, fears,coldness, reserve, hatred and betrayal will constantly hideunder the same false veil of politeness, under that much-touted urbanity which we owe to the enlightenment of ourtimes. The name of the Master of the Universe will nolonger be profaned by swearing, but insulted by blas-phemies that will not offend our scrupulous ears. Men willnot boast of their own merits, but belittle those of others.An enemy will not be crudely insulted, but adroitly slan-dered. National hatreds will die, but so will patriotism. Adangerous skepticism will take the place of the scorning ofignorance. Some excesses will be forbidden, some vicesdishonored, but others will be dignified with the name ofvirtues, and one must either have them or feign them. Letthose who want to praise the sobriety of the sages of ourtime do so; as for me, I see in it only a refinement ofintemperance that is as unworthy of my praise as their hyp-ocritical simplicity.

91. According to the tone and content of the passage,which statement best reflects the author’s opinion ofthe purpose of the sciences and the arts?

A. They are necessary for the safety and well-beingof mankind.

B. They interfere with responsive action and hon-est communication.

C. They reinforce the sense of “original liberty”that is present in all men.

D. They have molded our reactions into predictablebut essential forms.

30

GO ON TO THE NEXT PAGE.

5

10

15

20

25

30

35

40

45

50

55

60

65

70

75

03 MCAT FL Test3 06/27/2003 10:26 AM Page 30

Page 31: Full length 3

92. In the context of this passage, the “needs of the body”are to government and laws as:

A. the arts are to the “earthly powers” that “cherishtalents.”

B. the arts are to the “semblance of all the virtues.”

C. the arts are to chains that bind men into slavery.

D. the arts are to man’s sense of original liberty.

93. The author of this passage would most likely havethe greatest respect for a person who:

A. is deterred from acting on his own inclinationsby the needs of society.

B. does not dilute his natural reactions through theartifice of correct behavior.

C. exhibits his refinement through elaborate cloth-ing and conversation.

D. boasts of his own merit rather than belittling thatof others.

94. According to the author’s arguments, a society thatprizes its sciences and arts will:

A. respect urbanity.

B. scorn ignorance.

C. foster sincere friendships.

D. embrace simplicity.

95. According to the author, an “urbane” person is mostlikely to put the highest value on which of the fol-lowing characteristics?

A. simple, unadorned clothing.

B. reflection on one’s individual character.

C. polite relations in public discourse.

D. natural behavior and action.

96. Based on the opinions professed in the passage, theauthor would most likely believe that “well-basedconfidence” (line 59) would most likely arise from:

A. the uncertainty of not knowing another’s truefeelings.

B. knowledge of the true content of another’s char-acter.

C. the parity between appearance and true virtue.

D. knowledge of the absence of truth in the “veil”of politeness.

97. Which of the following is given as a supportingexample for the author’s concept of the ideal man?

A. One who exhibits courteous behavior.

B. One who maintains a pleasant demeanor.

C. One who rejects ornamentation.

D. One who cultivates talent.

31

GO ON TO THE NEXT PAGE.

03 MCAT FL Test3 06/27/2003 10:26 AM Page 31

Page 32: Full length 3

Passage IV (Questions 98–104)

Today’s new urban Asia is just as sophisticated and inmany ways more exciting than Western cities. Urban Asianconsumers are knowledgeable, modern, and keen toembrace the global lifestyle. Young, urban Asians havegrown up accustomed to many things that originated in theWest. They have, for example, completely embraced pizza,some even claiming that it is a part of their heritage. Thestory is told about a young Singaporean boy who wastaken by his father to Rome. “Hey, look, Dad,” the littleboy exclaimed, “they have pizza here too!” On samplingthe product, the boy decided that it was not as good as theoriginal back home.

Nury Vitachi, who writes for the South China Post andthe Far Eastern Economic Review, describes the Asianmiddle-class phenomenon: “Executives in Asia havebecome rich at warp speed by taking full control of theirown lives. They invest a great deal of time in their work,they use strategy to scramble up the corporate ladder, andthey demand payment in cash—so they can make theirmoney work as hard as they do.”

Signs of affluence are everywhere, but don’t get car-ried away. Traveling around Asia, no matter how rich theAsians become, signs of their frugal nature are still appar-ent. And they are very cost-conscious. Shopkeepers inmany Asian cities, most notably in Hong Kong, demandpayments for discounted merchandise in cash instead ofplastic, and many Asians are accustomed to that. Mostpeople save the increases in their income, and many preferto put it into fixed or other income-generating assets.Stock, land, and property are their favorites.

Many affluent Asians still regard financial security asthe most important form of security, and they are confidentthat Asia is the place to be to achieve that. While manyhave begun to savor the good life, they are not letting goof their top priority of education for their children. Educa-tion is looked upon as the most important contributing fac-tor to success in life. And in many of Asia’s competitiveurban centers, there is a rush to acquire a second degreeand other forms of professional qualification to ensure per-sonal competitiveness in the workplace. There are extraor-dinary opportunities in Asia for education and trainingprograms from language to software programming.

Despite the rise in their assertiveness, Asians still lookto the United States and not so much to Europe for ideasand trends. In general, except for those in Hong Kong andJapan, they are not particularly concerned with being fash-ionable. For today’s Asia, Japan and Hong Kong are thesources of Asian fashion ideas, but as Asia becomes more

affluent, there is a great opportunity to develop an indige-nous fashion industry. For example, a huge market poten-tial exists in introducing new materials and simplified butfashionable designs for countries in tropical Asia with ahot humid climate throughout the year.

The population density and lack of space in urbanareas has prohibited Asians from exercising frequently andfew indulge in outdoor activities. This is changing. MostAsians consider themselves in good health. Comparedwith Americans, few are overweight—largely as a result oftheir Asian diet. But now health clubs are becoming popu-lar among younger Asians. Potential for indoor exerciseequipment holds great promise. It is also important to dressfor the gym, and younger Asians are serious about lookinggood, complete with makeup, sunglasses, designer exer-cise shoes and outfits, and a gym bag.

In trying to hit Asia’s moving targets, regardless ofwhat you are selling, it is a good idea to stick with marketdensity—not country by country, but, mostly, city by city.Asian markets can be a marketer’s dream in that their den-sities are among the highest in the world. Java, Indonesia’smain island, has 115 million people. On Nanjing Road,Shanghai’s busiest street, businesses are open twelve hoursa day almost every day of the year. More than 1.5 millionpeople visit the shops there and spend more than $50 mil-lion every day.

Someone said that you can only become rich if you sellto the rich. I would add that you can become rich faster ifyou sell to the new rich. For investors in the West, watchfor Western companies that are preparing a big push inAsia. The world has not yet seen anything like it before,and you can reap handsome dividends if you back thosestocks that are going eastward.

98. According to the author, many Asians view educa-tion as:

A. the most fulfilling aspect of life.

B. a sector with tremendous growth opportunity.

C. the foremost step toward personal success.

D. an indicator of a family’s wealth and breeding.

32

GO ON TO THE NEXT PAGE.

5

10

15

20

25

30

35

40

45

50

55

60

65

70

75

80

03 MCAT FL Test3 06/27/2003 10:26 AM Page 32

Page 33: Full length 3

99. It can be inferred from the passage that:

A. the Asian fashion market is already saturatedwith indigenous designs and manufacturers.

B. credit cards are gaining acceptance slowly as aform of payment for merchandise in Asia.

C. the comparative lack of space in urban Asia hasled to increased health problems.

D. competitive professionals often pursue addi-tional education and training.

100. Suppose that Asian urban centers have seen the con-struction of many new hiking and nature trails. Howwould this information affect the author’s observa-tion on Asians’ interest in health?

A. It supports the author’s claim that Asians areexercising more.

B. It refutes the claim that health clubs are becom-ing more popular.

C. It supports the claim that outdoor activities arebecoming more popular if it can be shown thatthese trails are heavily used.

D. It refutes the claim that Asians consider them-selves in good health if it can be shown thatthese trails are heavily used.

101. Based on the information presented in the passage,with which of the following statements would theauthor most likely agree?

A. Every Asian nation shows some evidence ofwealth.

B. Routine exercise keeps many Asians in primephysical condition.

C. Many Asian cities are large enough to be con-sidered independent markets.

D. Asia’s markets have recently been flooded withAmerican goods.

102. The quote by Vitachi (lines 15–20) is used by theauthor to illustrate that:

A. Asians have an economic environment muchdifferent than the rest of the global economy.

B. Asian executives are becoming wealthy byadopting patterns similar to Western executives.

C. Asia is developing a middle class that has char-acteristics similar to those of Westerners.

D. Asian executives’ efforts to become rich areproving fruitful.

103. Which of the following is NOT directly supported bythe passage?

A. Asians have already assimilated many Westerncustoms and fashions.

B. Future investment in Asian markets will be themost lucrative action for Western manufacturers.

C. Urban professionals in Asia consider educationas beneficial to their careers.

D. The affluence of many Asians is not always evi-dent in their spending patterns.

104. In the passage, the author does all of the followingEXCEPT:

A. resolve a seeming economic paradox.

B. offer an interpretation of an observed trend.

C. project consequences of current developments.

D. make the information presented relevant to thereader.

33

GO ON TO THE NEXT PAGE.

03 MCAT FL Test3 06/27/2003 10:26 AM Page 33

Page 34: Full length 3

Passage V (Questions 105–111)

…[TV Guide’s] immediate concern was the televisionquiz show scandal, which had reached its climax twoweeks earlier when Charles Van Doren, the appealingyoung man who’d taught viewers the value of learningwhile winning big on MCA’s Twenty-one, stood before aHouse committee and admitted he was a fraud. But theissue went well beyond rigged quiz shows. The charge wasthat through their stranglehold on talent, MCA andWilliam Morris monopolized the medium to the detrimentof their clients, the industry, and the public at large. Thiswas why the Justice Department had launched a secretinvestigation of both agencies more than two years before.

The Morris Agency had started the quiz show vogue in1955, when it packaged The $64,000 Question for Revlonand sold it to CBS. While the show won praise for its “edu-cational” nature, the real source of its appeal was in itscrapshoot format—the idea that once contestants’ win-nings hit the $32,000 mark, they had to decide whether togo double or nothing on the final, $64,000 question, orplay it safe and go home. The response was tremendous.Within weeks, the show knocked I Love Lucy out of thenumber-one slot in the ratings. Casinos in Vegas emptiedout when it went on the air. Bookies took odds on whetherthe first contestant to go for the big one—a marine captainwhose specialty was cooking—would get the answer right.(He did.) Revlon sold so much Living Lipstick that its fac-tory was unable to meet the demand.

The $64,000 Question quickly inspired imitators,among them an MCA package called Twenty-one. Basedon the card game, more or less, Twenty-one was a dismalfailure at first. “Do whatever you have to do,” the sponsorordered angrily, so the producers put the fix in. In Decem-ber 1956, when Charles Van Doren, a boyishly attractiveEnglish instructor at Columbia University, beat HerbStempel, a short, squat, nerdy grad at City College, VanDoren became the first intellectual hero of the televisionage. Honors and acclaim poured in—the covers of Time,letters by the hundreds, offers of movie roles and tenuredprofessorships and a regular guest spot on The TodayShow. But Herb Stempel didn’t like being told to lose,especially to some Ivy League snot. He went to the press.The DA’s office started to investigate. The walls began toclose in.

Meanwhile, the show’s producers agreed to sell therights to NBC for $2 million. One of them started to feelqueasy about selling the show without letting the networkknow the score, so he went to Sonny Werblin, MCA’s topman in New York, and asked his advice. Werblin, the manbehind such hits as The Ed Sullivan Show and The Jackie

Gleason Show, ran the television department as if it were afootball team coached by Attila the Hun. “Dan,” he askedthe producer, “have I ever asked you whether the show wasrigged?” No, he hadn’t. “And has NBC ever asked youwhether the show is rigged?” No, they hadn’t either.“Well,” Werblin concluded, “the reason that none of us hasasked is because we don’t want to know.”

And with good reason. Not only was Twenty-one anMCA package and Van Doren himself an MCA client;Werblin had a special relationship with NBC’s president,Robert Kintner. Kintner had been president of ABCuntil…ABC’s chairman forced him out in his determina-tion to move the network out of third place. MCA used itsinfluence to place him at NBC, where he provedan extremely pliant customer. In the spring of 1957, whenthe networks were putting together their schedules for thenext season, Werblin went to a meeting of NBC program-ming executives led by Kintner and his boss, RCA chair-man Robert Sarnoff. “Sonny, look at the schedule for nextseason,” Kintner said when he walked in, “here are theempty slots, you fill them.”

105. Suppose that the contestants on the television gameshows mentioned in the passage had not been sup-plied with answers. Which of the following conclu-sions would most likely be correct?

A. The Justice Department would have ended itsinvestigation into the television productionindustry.

B. Herb Stempel would have continued as cham-pion on Twenty-one.

C. The ABC chairman would not have removedRobert Kintner from the presidency.

D. Herb Stempel would not have gone to the pressshortly after December 1956.

34

GO ON TO THE NEXT PAGE.

5

10

15

20

25

30

35

40

45

50

55

60

65

70

03 MCAT FL Test3 06/27/2003 10:26 AM Page 34

Page 35: Full length 3

106. Which of the following is neither supported nor con-tradicted in the passage?

A. Charles Van Doren received assistance thatenabled him to win.

B. Sonny Werblin was the sole creator of The EdSullivan Show.

C. The $64,000 Question quickly dethroned I LoveLucy from the top of the television ratings.

D. Robert Kintner had worked with another net-work before NBC.

107. The information in the passage would support thefollowing claims EXCEPT:

A. Werblin had an influential role at NBC.

B. Before 1955, quiz shows did not predominate intelevision ratings.

C. While a contestant on Twenty-one, Charles VanDoren was aware that he was cheating.

D. Concern over fraud in television game showswas the primary reason why the Justice Depart-ment investigated MCA.

108. Based on the passage, which of the following mustNOT be true?

A. Sonny Werblin was a difficult and exacting manto work with.

B. Herb Stempel refused to offer information to themedia concerning his appearances on Twenty-one.

C. The $64,000 Question was rigged.

D. Sonny Werblin was unfamiliar with RobertKintner before he was made president of NBC.

109. The author’s description of Kintner as “an extremelypliant customer” (line 64):

A. reinforces his description of Werblin as anexacting taskmaster.

B. supports his contention that NBC lacked strongleadership.

C. indicates the degree of MCA’s influence overthe network.

D. contradicts the implication that NBC knew theirquiz show was rigged.

110. The quote by Werblin (lines 55–56) is offered pri-marily as support of which of following contentions?

A. Werblin did not believe that quiz-show riggingwas in the best interest of the public.

B. Werblin was an ambitious person who soughtadmiration for his successes.

C. Werblin was a ruthless executive who was influ-ential with the William Morris Agency.

D. Werblin was a shrewd character who knew theconsequences of knowingly promoting a riggedgame show.

111. According to the passage, which of the following aretrue statements?

I. A correlation between successful contes-tants and successful sponsors exists in thetelevision industry.

II. Most game shows in the 1950s wererigged.

III. Van Doren’s quiz-show success providedhim with further opportunity in his aca-demic career.

A. I only

B. II only

C. III only

D. None of the above

35

GO ON TO THE NEXT PAGE.

03 MCAT FL Test3 06/27/2003 10:26 AM Page 35

Page 36: Full length 3

Passage VI (Questions 112–116)

…Squeaking sand produces sounds with very high fre-quencies—between 500 and 2,500 hertz, lasting less thana quarter of a second. The peals are musically pure, oftencontaining four or five harmonic overtones. Booming sandmakes louder, low-frequency sounds of 50 to 300 hertz,which may last as long as 15 minutes in larger dunes(although typically they last for seconds or less). In addi-tion, they are rather noisy, containing a multitude of nearbyfrequencies. Booms have never been observed to containmore than one harmonic of the fundamental tone.

These dramatic differences once led to a consensus thatalthough both types of sand produce acoustic emissions,the ways in which they do so must be substantially differ-ent….In the late 1970s, however, Peter K. Haff, then at theCalifornia Institute of Technology, produced squeaks inbooming sand, suggesting a closer connection between thetwo.

Both kinds of sand must be displaced to make sounds.Walking on some sand, for example, forces the sand under-foot to move down and out, producing squeaks. In the caseof booming sand, displacement occurs during avalanches.It is within the avalanche that sound begins and where theanswers must be hiding.

Before an avalanche can occur, winds must build adune up to a certain angle, usually about 35 degrees for drydesert sand. Once an angle is achieved, the sand on the lee-ward side of the dune begins to slump. Intact layers of sandslip over the layers below, like a sheared deck of cards. Atthe same time, the individual grains in the upper layerstumble over the grains underneath, momentarily fallinginto the spaces between them and bouncing out again tocontinue their downward journey. Their concerted up-and-down motion is believed to be the secret source of sound.Fully developed avalanches, in which sliding plates ofsand remain intact for most of their motion, have the great-est acoustic output. In some places, where large amountsof sand are involved, booming can be heard up to 10 kilo-meters away.

Because it is caused by large volumes of shearing sand,the roaring is also loud. In fact, sounds made by boomingsand can be nearly deafening, and the vibrations causingthem can be so intense that standing in their midst is nearlyimpossible.

A good place to start in exploring the vibrational prop-erties of sand is with the grains themselves. The meandiameter of most sand grains, whether acoustically activeor not, is about 300 microns. Usually the grains in a boom-

ing dune are very similar in size, especially near the lee-ward crest, where the sound most often originates; suchuniformity allows for more efficient shearing. Otherwise,the smaller grains impede the smooth motion of the largerones.

Similar sizes do not alone allow sand to boom. On thecontrary, the booming sands of Korizo and Gelf Kebib,also in Libya, feature an uncharacteristically broad rangeof particle sizes. Moreover, silent dune sand often containsgrains somewhat similar to nearby booming sand.

Grains of booming sand also tend to have uncommonlysmooth surfaces, with protrusions on the scale of meremicrons. Booming dunes are often found at the downwindend of large sand sources; having bounced and rolledacross the desert for long distances, the sand grains inthese dunes are usually highly polished. Over time a graincan also be polished by repeated shifts within a movingdune. And squeaking sand as well tends to be exception-ally smooth….

…Another important factor is humidity, because mois-ture can modify the friction between grains or cause sandto clump together, thus precluding shearing. Sounds occurin those parts of the dune that dry the fastest. Precipitationmay be rare in the desert, but dunes retain water withremarkable efficiency. Sand near the surface dries quickly,however, and sand around a dune’s crest tends to dry thefastest.

112. Which of the following characteristics is most help-ful in differentiating booming sand from squeakingsand?

A. duration of the sound

B. frequency of the sound

C. size of the sand grains

D. smoothness of the sand grains

36

GO ON TO THE NEXT PAGE.

5

10

15

20

25

30

35

40

45

50

55

60

65

70

03 MCAT FL Test3 06/27/2003 10:26 AM Page 36

Page 37: Full length 3

113. Booming occurs mostly in big dunes deep in thedesert. All of the following, if true, are factors thatmay plausibly account for this EXCEPT:

A. The grains need to be carried over large dis-tances by wind so they have a chance to be pol-ished down.

B. The sounds created by the booming dunes needa large distance to dissipate.

C. A relatively large sand source is needed sodunes can build up to the requisite volumebefore an avalanche occurs.

D. The dunes need to be far removed from anysource of moisture.

114. According to information presented in the passage,which of the following is true of all booming dunes?

A. The dune needs to build up to a certain criticalangle.

B. The dune is composed of sand grains of similarsize.

C. The dune is composed of exceptionally roughsand grains.

D. The dune needs to be dried for a while by windsto have moisture removed.

115. A dune is found to be comprised of smooth grains andproduces sounds that last less than a second. What fur-ther information, if any, would enable one to unam-biguously characterize the sand as squeaking sand?

A. The grains are not uniform in size.

B. The sound is generated by walking upon itrather than through avalanches.

C. The information given is already sufficient toconclude that the sand is squeaking sand.

D. The sand cannot possibly be squeaking sandfrom the information given.

116. Which of the following discoveries would give themost support to the hypothesis that squeaking sandand booming sand differ only in the mechanism bywhich the sounds are produced?

A. Avalanches can be induced in squeaking sand.

B. Squeaking sand can be made to generate boom-ing sounds.

C. Booming sounds made by dunes can be gener-ated by mechanisms other than avalanches.

D. Smooth grains are not a requirement for boom-ing sand.

37

GO ON TO THE NEXT PAGE.

03 MCAT FL Test3 06/27/2003 10:26 AM Page 37

Page 38: Full length 3

Passage VII (Questions 117–124)

When Gwendolyn Brooks published her first collec-tion of poetry A Street In Bronzeville in 1945 most review-ers recognized Brooks’ versatility and craft as a poet. Yet,while noting her stylistic successes few of her contempo-raries discussed the critical question of Brooks’ relation-ship to the Harlem Renaissance. How had she addressedherself, as a poet, to the literary movement’s assertion ofthe folk and African culture, and its promotion of the artsas the agent to define racial integrity?

The New Negro poets of the Harlem Renaissanceexpressed a deep pride in being Black; they found reasonsfor this pride in ethnic identity and heritage; and theyshared a common faith in the fine arts as a means of defin-ing and reinforcing racial pride. But in the literal expres-sion of this impulse, the poets were either romantics, orrealists and, quite often within the same poem, both. Therealistic impulse, as defined best in the poems of McKay’sHarlem Shadows (1922), was a sober reflection uponBlacks as second class citizens, segregated from the main-stream of American socio-economic life, and largelyunable to realize the wealth and opportunity that Americapromised. The romantic impulse, on the other hand, asdefined in the poems of Sterling Brown’s Southern Road(1932), often found these unrealized dreams in the collec-tive strength and will of the folk masses.

In comparing the poems in A Street in Bronzeville withvarious poems from the Renaissance, it becomes apparentthat Brooks brings many unique contributions to bear onthis tradition. The first clue that A Street In Bronzevillewas, at its time of publication, unlike any other book ofpoems by a Black American is its insistent emphasis ondemystifying romantic love between Black men andwomen. During the Renaissance, ethnic or racial pride wasoften focused with romantic idealization upon the Blackwoman. A casual streetwalker in Hughes’ poem, “WhenSue Wears Red,” for example, is magically transformedinto an Egyptian Queen. In A Street In Bronzeville, thisromantic impulse runs headlong into the biting ironies ofracial discrimination. There are poems in which Hughes,McKay and Brown recognize the realistic underside ofurban life for Black women. But for Brooks, unlike theRenaissance poets, the victimization of poor Black womenbecomes not simply a minor chord but a predominanttheme.

…Brooks’ relationship with the Harlem Renaissancepoets, as A Street in Bronzeville ably demonstrates, washardly imitative. As one of the important links with theBlack poetic tradition of the 1920s and 1930s, she enlargedthe element of realism that was an important part of the

Renaissance world-view. Although her poetry is often con-ditioned by the optimism that was also a legacy of theperiod, Brooks rejects outright their romantic prescriptionsfor the lives of Black women. And in this regard, sheserves as a vital link with the Black Arts Movement of the1960s that, while it witnessed the flowering of Blackwomen as poets and social activists as well as the rise ofBlack feminist aesthetics in the 1970s, brought about acurious revival of romanticism in the Renaissance mode.

117. Which of the following best expresses the main ideaof the passage?

A. The evolution of realism in Black women’spoetry can be traced from Gwendolyn Brooks tothe present day.

B. Gwendolyn Brooks’ first poems were unique inthe context of early twentieth-century poetry.

C. Contemporary scholars misinterpreted the cru-cial issue of Gwendolyn Brooks’ relationship tothe Harlem Renaissance.

D. Gwendolyn Brooks’ poetry brought a newemphasis on the realistic elements of theHarlem Renaissance tradition.

118. According to the passage, the poems in A Street inBronzeville are similar to the poems in Harlem Shad-ows because they each:

A. portray Black women in early twentieth-centuryAmerica as resourceful individuals who wereable to make successes of themselves.

B. influenced the poetry and social activism ofBlack women poets during the Black ArtsMovement of the 1960s.

C. are based entirely on the romantic impulse of theNew Negro poets of the Harlem Renaissance.

D. illustrate the grim realities of suffering and dis-crimination faced by Black Americans in earlytwentieth-century America.

38

GO ON TO THE NEXT PAGE.

5

10

15

20

25

30

35

40

45

50

55

03 MCAT FL Test3 06/27/2003 10:26 AM Page 38

Page 39: Full length 3

119. The passage suggests that the author would be mostlikely to agree with which one of the following state-ments about the poetry of the Harlem Renaissance?

A. The movement was inspired by a revival of folkand African culture.

B. The Harlem Renaissance poets portrayed acommon heritage in different ways.

C. The movement has been widely criticized for itslack of realism.

D. Brooks was more technically accomplished thanother Harlem Renaissance poets.

120. The author most probably mentions Hughes’ poem“When Sue Wears Red” (line 35–36) in order to:

A. prove that Brooks had not simply imitated theHarlem Renaissance poets.

B. highlight the critical role of the imagination inthe creative process.

C. contrast the irony of Brooks’ poetry with thenaiveté of earlier poetry.

D. provide an example of the romanticized por-trayal of Black women.

121. With which one of the following statements about AStreet in Bronzeville would the author most likelyagree?

A. It further developed the realistic impulse of theHarlem Renaissance tradition.

B. It was marred by an optimism inherited fromHarlem Renaissance poetry.

C. It completely rejected the poetic conventions ofthe Harlem Renaissance.

D. It was the first significant work of a Black fem-inist writer.

122. According to the passage, critics praised the qualityof Brooks’ first collection of poetry but:

A. rejected her description of the plight of poorBlack women in urban America.

B. failed to consider the links between her workand the work of earlier Black poets.

C. assumed incorrectly that she had borrowedmany ideas from the poems of Sterling Brown.

D. argued that she had neglected to demystifyromantic love between Black men and women.

123. Which of the following would best complete the lastparagraph of the passage?

A. For many readers, however, Brooks will best beremembered for her virtuosity in poetic technique.

B. In many ways, Brooks’ poetry owes more to theinfluence of the Black Arts movement than tothe poets of the Harlem Renaissance.

C. For while poets of the Black Arts movementwould often idealize their culture, their workwas tempered by realism.

D. But while her importance for later movements isestablished, Brooks’ relationship to the HarlemRenaissance remains open to question.

124. Suppose that a recently-discovered collection ofGwendolyn Brooks’ poems contained female protag-onists that embodied the ideal woman. This informa-tion would:

A. support the author’s contention that womenpoets were self-serving.

B. negate the author’s view that black poets pre-sented women and men with inequality.

C. contradict the author’s opinion that GwendolynBrooks allowed readers to experience a moreaccurate description of the modern Black woman.

D. neither support nor contradict the author’s claimthat Brooks served as an integral link betweenHarlem Renaissance poets and the Black ArtsMovement poets.

39

GO ON TO THE NEXT PAGE.

03 MCAT FL Test3 06/27/2003 10:26 AM Page 39

Page 40: Full length 3

Passage VIII (Questions 125–131)

…Until last year many people—but not mosteconomists—thought that the economic data told a simpletale. On one side, productivity—the average output of anaverage worker—was rising. And although the rate of pro-ductivity increase was very slow during the 1970’s andearly 1980’s, the official numbers said that it had acceler-ated significantly in the 1990’s. By 1994 an averageworker was producing about 20 percent more than his orher counterpart in 1978.

On the other hand, other statistics said that real, infla-tion-adjusted wages had not been rising at anything likethe same rate. In fact, some of the most commonly citednumbers showed real wages actually falling over the last25 years. Those who did their homework knew that thegloomiest numbers overstated the case….Still, even themost optimistic measure, the total hourly compensation ofthe average worker, rose only 3 percent between 1978 and1994….

…But now the experts are telling us that the wholething may have been a figment of our statistical imagina-tions….a blue-ribbon panel of economists headed byMichael Boskin of Stanford declared that the ConsumerPrice Index [C.P.I.] had been systematically overstatinginflation, probably by more than 1 percent per year for thelast two decades, mainly failing to take account of changesin the patterns of consumption and improvements in prod-uct quality….

…The Boskin report, in particular, is not an officialdocument—it will be quite a while before the Governmentactually issues a revised C.P.I., and the eventual revisionmay be smaller than Boskin and his colleagues propose.Still, the general outline of the resolution is pretty clear.When all the revisions are taken into account, productivitygrowth will probably look somewhat higher than it didbefore, because some of the revisions being proposed tothe way we measure consumer prices will also affect theway we calculate growth. But the rate of growth of realwages will look much higher—and so it will now beroughly in line with productivity, which will therefore rec-oncile numbers on productivity and wages with data thatshow a roughly unchanged distribution of income betweencapital and labor. In other words, the whole story aboutworkers not sharing in productivity gains will turn out tohave been based on a statistical illusion.

It is important not to go overboard on this point. Thereare real problems in America, and our previous concernswere by no means pure hypochondria. For one thing, itremains true that the rate of economic progress over the

past 25 years has been much slower than it was in the pre-vious 25. Even if Boskin’s numbers are right, the incomeof the median family—which officially has experiencedvirtually no gain since 1973—has risen by only about 35percent over the past 25 years, compared with 100 percentover the previous 25. Furthermore, it is quite likely that ifwe “Boskinized” the old data—that is, if we tried to adjustthe C.P.I. for the 50’s and 60’s to take account of changingconsumption patterns and rising product quality—wewould find that official numbers understated the rate ofprogress just as much if not more than they did in recentdecades….

…Moreover, while workers as a group have sharedfully in national productivity gains, they have not done soequally. The overwhelming evidence of a huge increase inincome inequality in America has nothing to do with priceindexes and is therefore unaffected by recent statisticalrevelations. It is still true that families in the bottom fifth,who had 5.4 percent of total income in 1970, had only 4.2percent in 1994; and that over the same period the share ofthe top 5 percent went from 15.6 to 20.1. And it is still truethat corporate C.E.O.’s, who used to make about 35 timesas much as their employees, now make 120 times as muchor more….

…While these are real and serious problems, however,one thing is now clear: the truth about what is happeningin America is more subtle than the simplistic morality playabout greedy capitalists and oppressed workers that somany would-be sophisticates accepted only a few monthsago. There was little excuse for buying into that simplisticview then; there is no excuse now….

125. Which of the following can be inferred as the bestdescription of the Boskin report?

A. A document that clarifies the existing percentagedistribution of income to labor and to capital.

B. A document that exposes the fact that the C.P.I.has overstated inflation for the past twenty years.

C. A document that reveals that worker productivityhas been overstated over the last twenty years.

D. A document that reveals that wages have signif-icantly decreased over the last twenty years.

40

GO ON TO THE NEXT PAGE.

5

10

15

20

25

30

35

40

45

50

55

60

65

70

75

03 MCAT FL Test3 06/27/2003 10:26 AM Page 40

Page 41: Full length 3

126. What is the effect of revising the C.P.I. on the calcu-lation of labor’s percentage share of nationalincome?

A. A greater disparity will be apparent between thehighest and lowest incomes.

B. The percentage share will appear the same.

C. The percentage share will appear smaller.

D. The percentage share will appear larger.

127. According to the passage, “Boskinization” adjuststhe C.P.I. by:

A. increasing wages and decreasing productivity toreconcile the present disparity.

B. taking into account technology’s role in animproved efficiency.

C. reassessing consumption patterns and quality ofproduct.

D. evaluating the inequalities in various levels ofincomes.

128. The Boskin report does all of the followingEXCEPT:

A. Reveals that the C.P.I. was inaccurate.

B. Reconciles the present disparity between pro-ductivity levels, wage levels, and the percentageof labor’s share in national income.

C. Reveals the reasons for the increasing disparitybetween the highest and lowest income earners.

D. Helps clarify economic progress in the 1950sand 1960s.

129. It can be inferred from the passage that in the 1950sand 1960s:

A. workers accounted for approximately the samepercentage of national income as in 1994.

B. workers experienced more substantial yearlypay increases than did workers in the 1970’s and1980’s.

C. workers’ wages, according to a revised C.P.I.,increased at a rate higher than economicprogress.

D. workers’ incomes accurately reflected theperiod’s economic progress.

130. Which of the situations below best reflects publicperception regarding the economy prior to the releaseof Boskin’s report?

A. Productivity has increased at a much higher ratethan employee compensation since 1970.

B. The rate of growth of productivity was approxi-mately that of wages.

C. The distribution of income to labor has radicallychanged over the last fifteen years.

D. Economic progress has been steady since 1945.

131. The author mentions the figures in paragraph six(lines 67–72) in order to show that:

A. the total productivity of America has not seen asignificant increase since the 1970’s.

B. each American worker’s productivity is directlyproportional to overall national productivitygains.

C. the income inequality in America is a problemthat is not eliminated by revision of the priceindex.

D. Boskin’s report is unable to explain the discrep-ancy between productivity growth and wageincreases.

41

GO ON TO THE NEXT PAGE.

03 MCAT FL Test3 06/27/2003 10:26 AM Page 41

Page 42: Full length 3

Passage IX (Questions 132–137)

For the last two decades many earth scientists havesupported the notion that the Mediterranean was once ahuge, dry desert, lying 3,000 meters below sea level. This“death valley” was thought to have existed at the end ofMiocene time, about 6 to 5.5 million years ago….

…From a geological point of view, the Mediterraneanis a tectonically mobile land-enclosed depression—small(about 3,000,000 square kilometers) in comparison to themajor world oceans…Immediately obvious on all charts isthe highly variable topography and relief of both theseafloor and adjacent borderland. The coastline is highlyirregular and continental shelves, though generally narrow,are well developed off the major river deltas (Nile, Rhone,Po, and Ebro). Moreover, the deep-sea basins and trencheshave distinctive relief, with basin plains ranging in depthfrom less than 1,000 meters to more than 4,000…Obser-vation that rocks dredged offshore are similar to those onland raised a fundamental concept—the key to under-standing Mediterranean history lies in the adjacentemerged land masses, and vice-versa….

…Early paleographic reconstructions showed that theonce-open communication with the Atlantic deterioratedduring the upper Miocene. Water-mass exchange contin-ued for a while in the Rif Strait, but then ceased com-pletely prior to the beginning of the Pliocene….

…High relief near what is now the Strait of Gibraltarserved as a barrier to the exchange of waters with theAtlantic. Exposed to a hot and dry climate, water evapo-rated and the then-dry basin elicited comparison with agigantic Death Valley…Microfossil studies suggested thatthe depth of the Mediterranean basin at these times hadbeen “deep.” Estimates suggested a dry seafloor as far as2,000 meters below ocean level… As a response to sud-denly lowered sea level, rivers feeding the Mediterraneanand canyons on the now-dry seafloor began a geologicallydramatic phase of erosion. Deep, Grand Canyon-likegorges of the Nile and Rhone rivers, presently buried onland, were apparently cut during a great drawdown ofwater—when the Mediterranean floor lay exposed 1,000meters or more below its present level…The sudden flood-ing through a gigantic waterfall at Gibraltar drowned theexposed basin floor. These falls would have been 1,000times bigger than Niagara Falls…This flooding event isrecorded by the Miocene Pliocene boundary, a time whenopen marine faunal assemblages were suddenly reintro-duced from the Atlantic….

…Geological theories usually fall at a glacial pace intoa sea of controversy, and this one is no exception. Today—

charging that proof for the theory is lacking—many scien-tists believe that the Med always contained saltwater, withonly the depth of the seafloor and the water being in ques-tion… Some of the tenets on which the theory was formu-lated are, if not defective, very seriously in question. Tointerpret their findings, a respectable number of geologistsstudying the surrounding emerged borderland as well assubsea sections indicate that alternative, more comprehen-sive concepts must be envisioned….

…It is not realistic to envision the Mediterraneanseafloor of about 5 million years ago as a desert at 3,000meters below present ocean level. Several years ago…theMediterranean [was compared] to a complex picture-puz-zle that comprises numerous intricate pieces, many ofwhich are already in place. A general image is emerging,although gaps in some areas of the picture remain fuzzyand indistinct.

132. All of the following are features of the “desert the-ory” EXCEPT:

A. Around 5.5 to 6 million years ago, the earth’sclimate was similar to that of modern day Earth.

B. The arid region compared to a “Death Valley”existed below sea level.

C. The start of the Pliocene period saw a waterfallat the Strait of Gibraltar that rivaled Niagara inits intensity.

D. During the Miocene, the Rif Strait served as achannel for the Atlantic waters to mix with theMediterranean.

133. It can be inferred from the passage that geologicaltheories tend to:

A. exist in the world of science with little contro-versy.

B. develop over short periods of time after multipleresearch efforts.

C. be disproved with the next trend in scientificthought.

D. evolve to incorporate the growing body ofknowledge derived from geological research.

42

GO ON TO THE NEXT PAGE.

5

10

15

20

25

30

35

40

45

50

55

60

65

03 MCAT FL Test3 06/27/2003 10:26 AM Page 42

Page 43: Full length 3

134. In the context of the passage, the term “emerged landmasses” (lines 19-20) refers to:

A. regions of land that surround the Mediterranean.

B. barriers that develop into waterfalls.

C. saline-concentrated soil that extracts water fromsurrounding tributaries.

D. continents that have come to touch one another.

135. All of the following are characteristics of the modernday Mediterranean EXCEPT that it:

A. has a variety of depths.

B. has a seafloor with a very regular and smoothtopography.

C. is land-enclosed and tectonically mobile.

D. contains well-developed continental shelves.

136. According to the passage, during the Mioceneperiod:

A. tectonic plates collided creating the massivewaterfall at the Strait of Gibraltar.

B. the composition and size of the Mediterraneandiffered greatly than the present.

C. rivers dried up as they filled the Mediterranean.

D. an abundance of flora and fauna existed that arenow extinct.

137. According to the author, which of the following is themost likely theory concerning the formation of theMediterranean?

A. Once a “Death Valley,” the Mediterranean basineventually filled with water to become a “sea.”

B. At the beginning of the Pliocene, an intensewaterfall connected the Atlantic Ocean to thedesert land mass that later became the Mediter-ranean.

C. The approaching Eurasian and African landmasses “pinched off” a large body of water thatdeveloped into the modern day Mediterranean.

D. None of the above

STOP. IF YOU FINISH BEFORE TIME IS CALLED,CHECK YOUR WORK.YOU MAY GO BACK TO ANYQUESTION IN THIS SECTION ONLY.

43

03 MCAT FL Test3 06/27/2003 10:26 AM Page 43

Page 44: Full length 3

03 MCAT FL Test3 06/27/2003 10:26 AM Page 44

Page 45: Full length 3

Writing SampleTime: 60 Minutes

2 Items, Separately Timed:30 Minutes Each

DO NOT BEGIN THIS SECTION UNTIL YOU ARE TOLD TO DO SO.

03 MCAT FL Test3 06/27/2003 10:26 AM Page 45

Page 46: Full length 3

WRITING SAMPLE

DIRECTIONS: This section is a test of your writing skills. Thesection contains two parts. You will have 30 minutes to completeeach part.

Your responses to the prompts given in the Writing Sample willbe written in the ANSWER DOCUMENT. Your response to Part1 must be written only on the answer sheets marked “1,” and yourresponse to Part 2 must be written only on the answer sheetsmarked “2.” You may work only on Part 1 during the first 30 min-utes of the test and only on Part 2 during the second 30 minutes.If you finish writing on Part 1 before the time is up, you mayreview your work on that part, but do not begin writing on Part 2.If you finish writing on Part 2 before the time is up, you mayreview your work only on Part 2.

Use your time efficiently. Before you begin writing a response, readthe assignment carefully and make sure you understand exactlywhat you are being asked to do. You may use the space below eachwriting assignment to make notes in planning your responses.

Because this is a test of your writing skills, your response to eachpart should be an essay composed of complete sentences andparagraphs, as well organized and clearly written as you canmake it in the allotted time. You may make corrections or addi-tions neatly between the lines of your responses, but do not writein the margins of the answer booklet.

There are six pages in your answer booklet to write yourresponses, three pages for each part of the test. You are notrequired to use all of the pages, but to be sure that you haveenough space for each essay, do not skip lines.

Essays that are illegible cannot be scored. In addition, essays thatare not written in English will not be scored.

46

03 MCAT FL Test3 06/27/2003 10:26 AM Page 46

Page 47: Full length 3

Part 1

Consider the following statement:

Education comes not from books but from practical experience.

Write a unified essay in which you perform the following tasks. Explain what you think the above statement means. Describe a specific situation in which books might educate students better than practical experience. Discuss what you think determines when practical experience provides a better education than books do.

47

03 MCAT FL Test3 06/27/2003 10:26 AM Page 47

Page 48: Full length 3

Part 2

Consider the following statement:

Developed nations have an obligation to provide aid to the underdeveloped nations of the world.

Write a unified essay in which you perform the following tasks. Explain what you think the abovestatement means. Describe a specific situation in which a developed nation might not be obligated toprovide aid to an underdeveloped nation. Discuss what you think determines when developed nationshave an obligation to provide aid to underdeveloped nations.

48

03 MCAT FL Test3 06/27/2003 10:26 AM Page 48

Page 49: Full length 3

Biological SciencesTime: 100 MinutesQuestions 138–214

DO NOT BEGIN THIS SECTION UNTIL YOU ARE TOLD TO DO SO.

03 MCAT FL Test3 06/27/2003 10:26 AM Page 49

Page 50: Full length 3

BIOLOGICAL SCIENCES

DIRECTIONS: Most of the questions in the BiologicalSciences test are organized into groups, with a descrip-tive passage preceding each group of questions. Studythe passage, then select the single best answer to eachquestion in the group. Some of the questions are notbased on a descriptive passage; you must also selectthe best answer to these questions. If you are unsure ofthe best answer, eliminate the choices that you knoware incorrect, then select an answer from the choicesthat remain. Indicate your selection by blackening thecorresponding circle on your answer sheet. A periodictable is provided below for your use with the questions.

50

GO ON TO THE NEXT PAGE.

1

H

1.0

2

He

4.0

3

Li

6.9

4

Be

9.0

5

B

10.8

6

C

12.0

7

N

14.0

8

O

16.0

9

F

19.0

10

Ne

20.2

11

Na

23.0

12

Mg

24.3

13

Al

27.0

14

Si

28.1

15

P

31.0

16

S

32.1

17

Cl

35.5

18

Ar

39.9

19

K

39.1

20

Ca

40.1

21

Sc

45.0

22

Ti

47.9

23

V

50.9

24

Cr

52.0

25

Mn

54.9

26

Fe

55.8

27

Co

58.9

28

Ni

58.7

29

Cu

63.5

30

Zn

65.4

31

Ga

69.7

32

Ge

72.6

33

As

74.9

34

Se

79.0

35

Br

79.9

36

Kr

83.8

37

Rb

85.5

38

Sr

87.6

39

Y

88.9

40

Zr

91.2

41

Nb

92.9

42

Mo

95.9

43

Tc

(98)

44

Ru

101.1

45

Rh

102.9

46

Pd

106.4

47

Ag

107.9

48

Cd

112.4

49

In

114.8

50

Sn

118.7

51

Sb

121.8

52

Te

127.6

53

I

126.9

54

Xe

131.3

55

Cs

132.9

56

Ba

137.3

57

La *

138.9

72

Hf

178.5

73

Ta

180.9

74

W

183.9

75

Re

186.2

76

Os

190.2

77

Ir

192.2

78

Pt

195.1

79

Au

197.0

80

Hg

200.6

81

Tl

204.4

82

Pb

207.2

83

Bi

209.0

84

Po

(209)

85

At

(210)

86

Rn

(222)

87

Fr

(223)

88

Ra

226.0

89

Ac †

227.0

104

Rf

(261)

105

Ha

(262)

106

Unh

(263)

107

Uns

(262)

108

Uno

(265)

109

Une

(267)

*

58

Ce

140.1

59

Pr

140.9

60

Nd

144.2

61

Pm

(145)

62

Sm

150.4

63

Eu

152.0

64

Gd

157.3

65

Tb

158.9

66

Dy

162.5

67

Ho

164.9

68

Er

167.3

69

Tm

168.9

70

Yb

173.0

71

Lu

175.0

90

Th

232.0

91

Pa

(231)

92

U

238.0

93

Np

(237)

94

Pu

(244)

95

Am

(243)

96

Cm

(247)

97

Bk

(247)

98

Cf

(251)

99

Es

(252)

100

Fm

(257)

101

Md

(258)

102

No

(259)

103

Lr

(260)

PERIODIC TABLE OF THE ELEMENTS

03 MCAT FL Test3 06/27/2003 10:26 AM Page 50

Page 51: Full length 3

Passage I (Questions 138–143)

Early experimentation on the single-celled organismAcetabularia led to important discoveries about the role ofthe nucleus in regulating cell function. Acetabularia is anenormous single cell with three distinct regions: a cap, aroot-like rhizoid, and a stalk which connects the two. Thefollowing experiments were conducted to study the devel-opment of the cell:

Experiment 1

The stalk of an Acetabularia was cut, fragmenting thecell. The fragment which included the cap died shortlyafterwards while the fragment containing the rhizoidregenerated to form a complete Acetabularia.

Experiment 2

The nucleus from Acetabularia mediterranea, whichhas a flat cap, was transplanted into Acetabularia crenu-lata, which has a tufted cap, following removal of theAcetabularia crenulata nucleus. The Acetabularia crenu-lata cap eventually assumed the flat shape.

Experiment 3

The nucleus of Acetabularia mediterranea wasremoved from the young cell before it first formed a cap.A normal cap formed several weeks later. The cell provedto be inviable and died shortly thereafter.

Experiment 4

A young Acetabularia was fractioned into a number ofportions before it first formed a cap. Several weeks later,both the portion containing the nucleus and the portioncontaining the apical tip of the stalk formed caps. Theother portions did not form caps.

138. One explanation for the results of Experiment 4 isthat the instructions for forming the cap are stored inthe apical tip of the stalk several weeks prior to stalkformation. Which of the following pieces of evidencebest supports this explanation?

A. Isolation of DNA coding for cap-inducing pro-teins from samples of Acetabularia taken sev-eral weeks prior to stalk formation.

B. Exposure of a young Acetabularia to ribonucle-ase, which cleaves RNA, blocks cap formation.

C. Exchange of nuclei between Acetabulariacrenulata and Acetabularia mediterranea leadsto formation of the cap associated with eachnucleus.

D. mRNA coding for cap-inducing proteins isfound to accumulate in the stalk apex.

139. Acetabularia is a(n):

A. virus.

B. prokaryote.

C. eukaryote.

D. bacterium.

140. Which of the following conclusions can be logicallydrawn from the fact that the Acetabularia segmentcontaining the rhizoid regenerated a complete andviable Acetabularia in Experiment 1?

A. The cell nucleus is located in the rhizoid.

B. Acetabularia reproduces by budding.

C. The Acetabularia cap is a vestigial structure.

D. Cap-coding mRNA is stored in the rhizoid.

51

GO ON TO THE NEXT PAGE.

03 MCAT FL Test3 06/27/2003 10:26 AM Page 51

Page 52: Full length 3

141. It can be inferred from the experiments in the passagethat development of the cap in Acetabularia is regu-lated by which of the following mechanisms?

A. Transcriptional regulation because mRNA in thecytoplasm lies dormant for several weeks beforecap formation occurs.

B. Translational regulation because mRNA is notproduced by the nucleus until it is required forcap production.

C. Translational regulation because mRNA in thecytoplasm lies dormant for several weeks beforecap formation occurs.

D. Neither transcriptional regulation nor transla-tional regulation.

142. The differences in cap structure between Acetabu-laria mediterranea and Acetabularia crenulata arecaused by differences in:

A. genotype.

B. phenotype.

C. genus.

D. phylum.

143. The mRNA sequence shown below is transcribedfrom which of the following DNA fragments?

5'-UGUAAUC-3' mRNA

A. 5'-GAUUACA-3'

B. 5'-ACATTAG-3'

C. 5'-CTAATGT-3'

D. 5'-GATTACA-3'

52

GO ON TO THE NEXT PAGE.

03 MCAT FL Test3 06/27/2003 10:26 AM Page 52

Page 53: Full length 3

Passage II (Questions 144–149)

Nitric oxide, NO, has recently been found to havewidespread physiological effects, acting as a major regula-tor in the nervous, cardiovascular, and immune systems.The production of NO in the body is regulated by specificNOS enzymes which exist in at least three different iso-forms—bNOS, eNOS, and macNOS. Each of these iso-forms differ in location and function and serve to mediatedifferent physiological responses to NO. Some physiolog-ical roles of NO have been demonstrated as follows:

I. In the central nervous system, NO productionis regulated by bNOS. Calcium ion concentra-tions of 200-400 nM in the central nervoussystem activate bNOS to catalyze the forma-tion of NO. NO exerts definite effects on brainfunction although its specific roles are not wellestablished. bNOS inhibitors have been foundto block the release of neurotransmitter frompresynaptic neurons. Excess levels of NO arealso thought to contribute to neurodegenera-tive disorders such as Alzheimer’s disease.

II. In the blood vessels, NO is produced by eNOSwhich is activated by Ca2+ concentrations of200-400 nM. NO acts as the major endoge-nous vasodilator in blood vessels. It diffusesinto smooth muscle cells and leads to musclerelaxation by stimulating cGMP formationthrough activation of guanylyl cyclase. Inaddition, NO regulates the vascular system byinhibiting platelet aggregation and adhesion.

III. The role of NO in the immune system is reg-ulated by macNOS through a pathway that isnot Ca2+ dependent. Rather, exposure tocytokines, including interleukin-1 and inter-feron-γ, leads to synthesis of large amountsof NO by activation of macNOS in responseto inflammatory stimuli. The NO producedplays a definitive role in the mediation of theactivities of macrophages and neutrophils.NO also acts to inhibit the mechanism ofviral replication.

144. Based on information in the passage, which of thefollowing is a possible effect of NO in the brain?

A. Inhibit the fusing of neurotransmitter vesicles atthe presynaptic membrane

B. Promote neurotransmitter release into thesynaptic cleft

C. Enhance vasoconstriction to maintain bloodpressure

D. Decrease neurotransmitter synthesis by thepresynaptic neuron

145. A “knock out” mouse with a mutant bNOS proteinwas generated by recombination techniques. Themutant protein was identical to the wild-type proteinexcept for the identity of amino acid 675; the mutatedbNOS has Tryptophan instead of Cysteine at position675. Which of the following is responsible for themutant protein?

A. Frame shift mutation

B. Single base-pair deletion

C. Point mutation

D. Nonsense mutation

146. A patient has accidentally ingested a toxin which actsas an eNOS inhibitor. According to the passage, theeffects of this toxin would most likely include:

A. decreased blood pressure.

B. increased immune response.

C. decreased urine production.

D. increased blood pressure.

53

GO ON TO THE NEXT PAGE.

03 MCAT FL Test3 06/27/2003 10:26 AM Page 53

Page 54: Full length 3

147. A patient with hypertension (high blood pressure) istreated with nitroglycerin, which spontaneouslybreaks down to give NO. This treatment will be:

A. effective because NO will activate eNOS.

B. effective because NO will lead to vasodilation.

C. ineffective because eNOS will not be activated.

D. ineffective because NO will build up, leading toneurotoxicity.

148. According to the passage, NO may counter viralinfection by all of the following mechanismsEXCEPT:

A. inducing white blood cells to engulf and destroyforeign agents.

B. inhibiting the synthesis of viral RNA.

C. denaturing viral protein coats in the interstitialfluid.

D. blocking viral release from infected cells.

149. Which of the following Ca2+ concentrations couldlead to activation of bNOS?

A. 3.2 × 10–7 M

B. 2.5 × 10–9 M

C. 1.9 × 10–7 M

D. 3.8 × 10–8 M

54

GO ON TO THE NEXT PAGE.

03 MCAT FL Test3 06/27/2003 10:26 AM Page 54

Page 55: Full length 3

Passage III (Questions 150–155)

Synthetic dyes constitute a commercially significantarea of organic chemistry. The color producing propertiesof these compounds are the result of highly delocalizedelectron systems giving rise to electronic transitions whoseabsorptions occur in the visible region. Most commerciallyuseful dyes can be classified as one of three types—anthraquinones, azo dyes, or triarylmethyl salts. Examplesof each type are illustrated in Figure 1.

Figure 1

In order for a dye to be useful in the fabric industry, itmust have sufficient affinity for the polymeric fibers ofwhich the material is composed; the dye must not onlyimpart a color to the fabric, but must also do so in a rela-tively permanent manner (color fastness). Proper design ofsynthetic polymers requires the placement of acidic orbasic side chains along the polymer backbone such thatbinding sites are available for dying. Similarly, dyes mustbe produced not only with the appropriate color-producingstructure, but also with an affinity for the fabric in ques-tion. The structural units of several common syntheticfibers are shown in Figure 2.

Figure 2

150. Certain natural protein fibers such as silk or wool canbe treated with aqueous base, then with solutionscontaining cationic dyes such as malachite green toproduce color fast yarns. The most likely explanationfor the affinity of malachite green for silk or wool viathis process is that:

A. many of the R groups on the amino acids ofwhich these fibers are composed contain COOHgroups.

B. very few of the R groups on the amino acids ofwhich these fibers are composed contain OHgroups.

C. the aqueous base hydrolyzes some of the pep-tide linkages in these fibers.

D. the aqueous base neutralizes the cationic dye.

151. Dacron belongs to which of the following generalclassifications?

A. Polyamide

B. Polyester

C. Polyurethane

D. Polypeptide

152. Cotton is a natural fiber composed of cellulose, apolymer of glucose. Which of the compounds shownin Figure 1 would adhere to a cotton fiber via hydro-gen bonding?

A. Alizarin only

B. Malachite green only

C. Alizarin and aniline yellow only

D. Malachite green, alizarin, and aniline yellow

NH(CH2)6NH C(CH2)4

O

C

O

nylon 66

NH(CH2)6NH C(CH2)8

O

C

O

nylon 610

CC6H4

O

C

O

O CH2CH2O

Dacron

O

OOH

HO

anthraquinone(alizarin)

N N NH2

azo dye(aniline yellow)

C+

(H3C)2N N(CH3)2triarylmethyl cation(malachite green)

55

GO ON TO THE NEXT PAGE.

03 MCAT FL Test3 06/27/2003 10:26 AM Page 55

Page 56: Full length 3

153. Nylon, Dacron, and many other synthetic fibers areproduced via condensation reactions. Which of thefollowing would be the best starting materials for theproduction of nylon 66?

A. cis-2-butenoic acid and 1,6-hexanediamine

B. butanedioic acid and 1,6-hexanediamine

C. n-hexanoic acid and 2,5-hexanediamine

D. hexanedioic acid and 1,6-hexanediamine

154. Azo dyes are synthesized via diazotization. Thesequence of diazotization involves first treating anaromatic amine with NaNO2 in acidic solution,thereby converting it to the corresponding diazoniumsalt (with the general formula ArN2X–), then reactingwith a second aromatic compound to form the azodye. What is the structure of the azo dye produced bythe reaction sequence below?

A.

B.

C.

D.

155. Hydrogenation of alizarin over a specific rhodiumcatalyst results in the complete saturation of all thecarbon-carbon bonds, but does not affect any otherfunctional groups. Which of the following effectsshould also be expected?

A. The hydrogenated product will no longer haveany affinity for Dacron fibers.

B. The hydrogenated product will no longer be sol-uble in aqueous base.

C. The hydrogenated product will no longer appearcolored.

D. The hydrogenated product will have a greatlyincreased affinity for nylon 610.

N N

NO2

H3CO

N N

H3CO

NO2

N N NO2

N N OCH2

NH2

O2N OCH3NaNO2

HCldiazonium salt

56

GO ON TO THE NEXT PAGE.

03 MCAT FL Test3 06/27/2003 10:26 AM Page 56

Page 57: Full length 3

Passage IV (Questions 156–162)

Hypoxia refers to a physiological condition in whichthe body lacks sufficient oxygen for normal cellular func-tioning. Prolonged hypoxia generally leads to an inhibitionof mental capacity and a reduction in the work capacity ofmuscle. Severe cases of hypoxia can lead to coma or evendeath. Depending on the cause, hypoxia can be classifiedinto four general types:

Hypoxic hypoxia is a type of hypoxia that occurs whenthe partial pressure of oxygen in the blood is too low. Forexample, climbers at high altitude, where the air containsless oxygen, might experience hypoxic hypoxia becausethe partial pressure of oxygen in the air inhaled is very low,leading to insufficient partial pressure of oxygen in theblood.

Anemic hypoxia describes a diminished ability of theblood to transport oxygen. Several factors can influencethe oxygen-carrying capacity of the blood. Primary causesof anemic hypoxia include a lower than normal number offunctional erythrocytes or an insufficient quantity ofhemoglobin, the oxygen-carrying molecules of the blood.Abnormal hemoglobin can also decrease the blood’scapacity to carry oxygen and lead to anemic hypoxia.

Ischemic hypoxia is caused by a decreased delivery ofblood to the tissues. Localized circulatory deficiencies,such as blood clots, and global circulatory deficiencies,such as heart failure, decrease the delivery of blood to thetissues, and can therefore cause ischemic hypoxia.

Histotoxic hypoxia results from the inability of cells toutilize the oxygen available in the blood. Causes of histo-toxic hypoxia include the poisoning of cellular enzymesinvolved in aerobic respiration, as well as the decreasedmetabolic capacity of the oxidative enzymes due to vita-min deficiency. Cyanide poisoning causes histotoxichypoxia by blocking the action of cytochrome oxidase inthe electron transport chain so that tissues cannot use oxy-gen even though it is available.

156. Cigarette smoking causes emphysema, a condition inwhich the net surface area of the lungs is greatlydecreased leading to a decrease in the diffusingcapacity of the lungs. Emphysema leads to whichtype of hypoxia?

A. Hypoxic hypoxia

B. Anemic hypoxia

C. Ischemic hypoxia

D. Histotoxic hypoxia

157. Exposure to high levels of radiation has been demon-strated to cause anemia. The most likely explanationis that radiation damages the:

A. blood vessels.

B. spleen.

C. thymus.

D. bone marrow.

158. If cyanide is radioactively-labeled and its positiontraced within the cell, it will most likely be found in:

A. the Golgi apparatus.

B. membrane bound vesicles.

C. the nucleus.

D. the mitochondria.

159. The passages of the respiratory tract which do notparticipate in gas exchange are called the physiolog-ical dead space. Compared to air in the alveoli, air inthe physiological dead space will have:

A. higher PCO2and higher PO2

.

B. higher PCO2and lower PO2

.

C. lower PCO2and higher PO2

.

D. same PCO2and same PO2

.

57

GO ON TO THE NEXT PAGE.

03 MCAT FL Test3 06/27/2003 10:26 AM Page 57

Page 58: Full length 3

160. Carbon monoxide binds to hemoglobin as shown inthe dissociation curve below. The dissociation curvefor oxygen is also shown. These dissociation curvesindicate that:

A. carbon monoxide binds to hemoglobin with thesame affinity as does oxygen.

B. carbon monoxide saturates hemoglobin at muchlower partial pressures than does oxygen.

C. oxygen will saturate hemoglobin more rapidlythan will an equal partial pressure of carbonmonoxide.

D. carbon monoxide forms covalent bonds tohemoglobin while oxygen is held by non-cova-lent interactions.

161. Hypoxia can often be treated by ventilation with pure

oxygen. The increased PO2in the alveoli will lead to

an increased PO2in the blood. Treatment with pure

oxygen is LEAST effective in treating which of the

following types of hypoxia?

A. Hypoxic hypoxia caused by hypoventilation

B. Anemic hypoxia caused by sickle cell anemia

C. Ischemic hypoxia caused by poor circulation

D. Histotoxic hypoxia caused by the disease beriberi

162. A patent foramen ovale occurs when an infant’s fora-men ovale does not close completely at birth. Basedon the information presented in the passage, this canlead to:

A. hypoxic hypoxia because much of the blood

does not reach the lungs to be oxygenated lead-

ing to a low PO2.

B. anemic hypoxia because the blood does notcarry enough oxygen.

C. ischemic hypoxia because the flow of blood isshunted away from the lungs.

D. none of the above.

100

0

50

75

25

% H

bsa

tura

tion

PCO (mmHg )

0 0.2 0.4

100

0

50

75

25

% H

bsa

tura

tion

PO2 (mmHg)

0 70 140

58

GO ON TO THE NEXT PAGE.

03 MCAT FL Test3 06/27/2003 10:26 AM Page 58

Page 59: Full length 3

163. Which of the following molecules would produce theNMR spectrum shown below?

A. C.

B. D.

164. Which of the following best illustrates the contractedstate of the sarcomere shown below?

A.

B.

C.

D.

H3C C CH3

Br

H

H3C C CH3

Br

Br

H3C C CBr3

Br

H

H3C C CBr3

Br

Br

4.0 3.0 2.0 1.0 05.06.0PPM ( δ )

Questions 163 through 167 are NOT basedon a descriptive passage.

59

GO ON TO THE NEXT PAGE.

03 MCAT FL Test3 06/27/2003 10:26 AM Page 59

Page 60: Full length 3

165. In the small intestine, most amino acids are trans-ported from the lumen of the gut into the epithelialcells against their concentration gradient. This move-ment of amino acids is coupled to the movement ofsodium ions down their concentration gradient in aform of transmembrane movement called:

A. simple diffusion.

B. facilitated diffusion.

C. secondary active transport.

D. primary active transport.

166. Which of the following functional groups are NOTfound on the side chains of any of the naturallyoccurring amino acids?

A. Hydroxyl

B. Methyl

C. Carboxylic acid

D. Aldehyde

167. When humans are submerged in water, the mam-malian dive reflex acts to alter circulation. Heart ratedecreases, blood flow to the extremities is reduced,and mean arterial blood pressure is increased. Theseaccomodations lead to:

A. decreased oxygen demand by the tissues.

B. increased heat retention by the body.

C. decreased partial pressure of oxygen in theblood.

D. increased venous return.

60

GO ON TO THE NEXT PAGE.

03 MCAT FL Test3 06/27/2003 10:26 AM Page 60

Page 61: Full length 3

Passage V (Questions 168–172)

In 1972, Georges Ungar reported the discovery of apeptide that appeared to transfer learning. Ungar’s claimwas based on experiments in which rats placed in a cham-ber with specially designed dark and light regions weretrained to avoid the dark regions of the chamber. Follow-ing their training, the rats were killed and brain extractswere prepared. These brain extracts were injected intonaive rats which were then observed to acquire the fear ofdarkness without training. Two hypotheses were proposedto explain these remarkable results:

Hypothesis 1

Ungar concluded that the extracts contained somechemical that transmitted the learned fear of darkness tothe naive rats. A fifteen amino-acid polypeptide was iso-lated from the brain extracts and sequenced. Ungarclaimed that this peptide, called scotophobin, was a chem-ical transmitter of learning. The peptide had the primarystructure shown below:

C-ser-asp-asn-arg-gln-gln-gly-lys-ser-ala-arg-gln-gly-gly-tyr-N

scotophobin

Hypothesis 2

Other researchers, who tested scotophobin but couldnot reproduce Ungar’s results, argued that scotophobin didnot transfer the learned fear of darkness. Instead, they sug-gested that scotophobin, which is structurally similar toACTH and vasopressin, acted to increase stress in the rats.Since stress increases sympathetic nervous activity, ratsinjected with scotophobin would become hyperactive andtend to spend less time in the dark regions of the experi-mental chamber. They argued that such stress responses inthe rats could be misinterpreted as a fear of darkness.Ungar’s claim was further weakened by chemical analysisin which both the scotophobin extracts which Ungar hadinjected into the naive rats and a sample of synthesizedscotophobin peptide were subjected to SDS polyacry-lamide gel electrophoresis, as shown in Figure 1.

Figure 1

168. Hydrolytic enzymes cleave polypeptides at specificamino acid residues. Which of the followinghydrolytic enzymes could be used to cleave scoto-phobin into three fragments?

Enzyme Sites of Cleavage Bond Cleaved

Trypsin Lys or Arg carboxy end

Clositripain Arg carboxy end

Chymotrypsin Phe, Trp, or Tyr carboxy end

Pepsin Asp, Glu, Leu, carboxy endPhe, Trp, or Tyr

A. Trypsin only

B. Trypsin or clositripain only

C. Clositripain or chymotrypsin only

D. Clositripain or pepsin only

169. In a follow-up experiment, researchers administeredscotophobin to rats in order to examine its physio-logical effects. Which of the following observationsof physiological effects in the rat would provide thebest support for Hypothesis 2?

A. Increase in urine volume

B. The rat pupils dilate causing the rat to preferdarkness and avoid light

C. Increase in heart rate

D. Dilation of arterioles regulating blood flow tothe digestive tract

top (–)(large molecules)

bottom (+)(small molecules)

A

B

C

D

Ungar'sExtract

Syntheticscotophobin

61

GO ON TO THE NEXT PAGE.

03 MCAT FL Test3 06/27/2003 10:26 AM Page 61

Page 62: Full length 3

170. Researchers isolated a polypeptide from the brains ofgoldfish trained to avoid darkness. This goldfish sco-tophobin was 15 amino acids long and differed fromrat scotophobin by one amino acid. The gene forgoldfish scotophobin must differ from that of rat sco-tophobin by:

A. one amino acid.

B. three codons.

C. at least one nucleotide.

D. at least three nucleotides.

171. Researchers were interested in purifying a secondprotein (protein X) from Ungar’s extract. The genesegment encoding protein X was believed to consistof thirty nucleotides. According to Figure 1, whichband could represent protein X?

A. A

B. B

C. C

D. D

172. The chemical analysis of Ungar’s extract most likelyweakened his claim that scotophobin transferred thelearned fear of darkness because it showed that:

A. the extract was not pure scotophobin.

B. the extract did not contain scotophobin.

C. scotophobin was not present in sufficient con-centrations to illicit a response.

D. scotophobin was similar in structure to otherproteins.

62

GO ON TO THE NEXT PAGE.

03 MCAT FL Test3 06/27/2003 10:26 AM Page 62

Page 63: Full length 3

Passage VI (Questions 173–178)

Morphine alkaloids derived from the opium poppyhave long been used as analgesics. Codeine, the methylether of morphine, is a naturally occurring alkaloid withmedicinal properties very similar to those of morphine.Thousands of derivatives of morphine have been synthe-sized and tested for their biological effects. For example,the diacylated derivative of morphine, heroin, is a highlyaddictive drug. Much effort has gone into understandinghow morphine and its derivatives function.

Figure 1

Studies have shown that certain common structuralfeatures of alkaloids are required for the compound toexhibit biological activity. These structural requirementsare summarized by the so called “morphine rule”:

1. an aromatic ring adjacent to a quaternary carbon;2. two carbons between this quaternary carbon and a

tertiary amine.

Demerol and methadone, shown in Figure 2, are twosynthetic alkaloids designed to satisfy the “morphine rule.”Synthetic alkaloids such as these have been found tomimic certain physiological properties of morphine and itsderivatives, and have found pharmacological applicationdue to other, more desirable biological effects. Methadonehas been used widely in the United States and GreatBritain as a treatment for heroin addiction; it reduces the

physical symptoms accompanying withdrawal withoutproducing many of the other effects of heroin.

Figure 2

173. One of the requirements of the “morphine rule” isthat an aromatic ring be attached to a quaternary car-bon in order for the molecule to be biologicallyactive. The quaternary carbon of any morphine-likesubstance must be:

I. a stereocenter

II. sp3 hybridized

III. sp2 hybridized

A. I only

B. II only

C. I and II only

D. I and III only

174. How many chiral carbons are there in morphine?

A. 4

B. 5

C. 6

D. 7

N

C6H5 CCH2CH3

OCH3

CH3

CH3

Methadone

N CH3

Meperidine(demerol)

HO

O

H

H

HO

N

HCH3

Morphine

H3CO

O

H

H

HO

N

HCH3

Codeine

O

H

HN

HCH3

Heroin

CO

O

CH3

CO

O

CH3

63

GO ON TO THE NEXT PAGE.

03 MCAT FL Test3 06/27/2003 10:26 AM Page 63

Page 64: Full length 3

175. Which of the following compounds would be mostlikely to have morphine-like biological effects?

A.

B.

C.

D.

176. Morphine can be reacted with 2 equivalents ofethanoyl chloride (acetyl chloride) to form heroin. Inthis reaction, the hydroxy groups of morphine func-tion as:

A. nucleophiles.

B. electrophiles.

C. leaving groups.

D. Lewis acids.

177. Hofmann elimination involves methylation of theamine nitrogen followed by elimination (E2). Which ofthe following represents a possible product of onesequence of Hofmann elimination on Meperidine(demerol)?

A.

B.

C.

D.

178. Researchers have discovered that the stereochemistryof the alkaloids is critical to their physiologicaleffects. For example, (–)-methadone produces pow-erful analgesic effects while (+)-methadone producesno effect. Which of the following is the most likelyexplanation for this difference in activity?

A. (+)-Methadone does not follow the “morphinerules.”

B. Alkaloids bind to stereospecific receptors.

C. Analgesic effects require the rotation of planepolarized light.

D. (–)-Methadone is the naturally occurring enantiomer.

N CH3

NH2

NCH3

CH3

N CH3

CH3

+

N

N CH3

HO

O

H

H

HO

NH

H

NHO

64

GO ON TO THE NEXT PAGE.

03 MCAT FL Test3 06/27/2003 10:26 AM Page 64

Page 65: Full length 3

Passage VII (Questions 179–185)

The hydrogens of alkanes have pKa values that areover 30 or 40. In contrast, the α-hydrogens of aldehydesand ketones have pKa values that range from 19 to 21.These fairly acidic α-hydrogens can be removed by strongbases to form anions called enolates. The enolate ions arestrongly stabilized by resonance. Protonation of the eno-late at oxygen produces an enol. Interconversion betweenthe keto and enol forms is called tautomerization and isillustrated in Figure 1. The keto form is usually highlyfavored.

Figure 1

Keto-enol tautomerization has some interesting conse-quences. For example, if a ketone is treated with acid orbase in a solvent of D2O (heavy water), all of the α−hydrogens will be exchanged for deuterium. This reactionis shown in Figure 2.

Figure 2

Another consequence of keto-enol tautomerization isthe racemization of chiral α-carbons. In the enol form, theα-carbon adopts a planar configuration and is no longerchiral. Tautomerization back to the ketone produces aracemic mixture of products. This is shown in Figure 3.

Figure 3

179. A scientist attempts to follow the progress of theα-deuteration shown in Figure 2 using proton NMR.Which of the following would be the best indicatorthat the reaction has proceeded to completion?

A. Singlet at 9.8 ppm

B. Quadruplet between 1.0 and 2.0 ppm

C. Doublet between 3.0 and 4.0

D. No signal

180. The IUPAC name for the reactant in Figure 3 is:

A. (R)-3-methyl-2-pentanone

B. (R)-3-ethyl-2-butanone

C. (R)-3-ethyl-3-methyl-propanal

D. (R)-2,2-diethyl-propanone

181. Which of the following ketones will have the mostacidic α-hydrogen:

A. C.

B. D.

182. The two products formed by the racemization reac-tion in Figure 3 could best be described as:

A. enantiomers

B. diastereomers

C. conjugate acid and base

D. same compound

Cl3C

O

CCl3Cl3C

O

CH3

H2NH2C

O

CH3H3C

O

CH3

C

O

CH3

C2H5

H

H3C

C

O

CH3

C2H5

H3C

H

C

O

CH3

C2H5

H

H3C

EtO-, EtOH

R

S

R

H3C

O

CH3 D3C

O

CD3

D2O, DO-

O OH

keto enol

65

GO ON TO THE NEXT PAGE.

03 MCAT FL Test3 06/27/2003 10:26 AM Page 65

Page 66: Full length 3

183. The aldol condensation proceeds through formationof an enolate anion. This is followed by nucleophilicattack by the enolate ion on the carbonyl carbon ofthe original aldehyde. Which of the following reac-tants would lead to the product indicated below?

A.

B.

C.

D.

184. All of the following reactions will lead to formationof an aldehyde EXCEPT:

A.

B.

C.

D.

185. Which of the following best represents the enolateion intermediate of the racemization reaction shownin Figure 3?

A. C.

B. D.

δ−

δ−

C

O

CH3

C2H5

H2C

δ−δ−

δ−C

O

CH2

C2H5

H

H3C

O

CH3

C2H5

H3C

δ−δ−O

CH3

C2H5

H3C

δ+δ+

C CHR1. Dicyclohexylborane

2. H2O2

OH KMnO4

OH

H

PCC

O

Cl

H2, Pd-BaSO4

Quinoline

O

H

O

H

O

Cl

O

O

H

HO

aldehydealdol condensation

product

66

GO ON TO THE NEXT PAGE.

03 MCAT FL Test3 06/27/2003 10:26 AM Page 66

Page 67: Full length 3

Passage VIII (Questions 186–191)

Artificial kidneys have been used for almost 50 yearsto treat patients with different forms of renal failure. Theartificial kidney (dialysis machine) removes unwantedsubstances from the blood by diffusion. A patient’s bloodis passed through channels bounded by a porous, semi-per-meable membrane that allows the free diffusion in bothdirections of all plasma constituents except the plasmaproteins. Erythrocytes and other cellular components ofblood cannot pass through the membrane. The other sideof the membrane is exposed to the dialyzing fluid whichcarries away the unwanted materials.

If the concentration of a material in the blood is greaterthan in the dialyzing fluid, there will be a net flow of thematerial from the plasma to the dialyzing fluid. If the con-centration of a material in the blood is less than in the dia-lyzing fluid, there will be a net flow of the material fromthe dialyzing fluid into the blood. The composition of nor-mal plasma, plasma in an individual suffering renal failure,and dialyzing fluid are shown in Table 1.

Constituent Normal Plasma w/ DialyzingPlasma renal Fluid

failure(mEq/L) (mEq/L) (mEq/L)

Na+ 142 142 133

K+ 5 7 1.0

Cl– 107 107 105

HCO3– 27 14 35.7

Urate 0.3 2 0

Constituent Normal Plasma w/ DialyzingPlasma renal Fluid

failure(mg/dl) (mg/dl) (mg/dl)

Glucose 100 100 125

Urea 26 200 0

Creatinine 1 6 0

Table 1

Dialysis replaces some functions of the kidneys andattempts to correct the effects of renal failure. For exam-ple, patients with renal failure develop acidosis due to abuildup of metabolically produced acids in the circulation.Without dialysis, the pH of the blood will drop and comamay occur. Dialyzing fluid contains a relatively high con-centration of bicarbonate which diffuses into the circula-tion and neutralizes the acid.

186. In order to prevent the net movement of waterbetween the blood and the dialyzing fluid, the dia-lyzing fluid:

A. is hypoosmotic to blood.

B. is isoosmotic to blood.

C. contains a higher concentration of solutes thanblood.

D. contains hydrophilic proteins.

187. Which of the following provides the best explanationfor the urea plasma concentration in individuals withrenal failure?

A. Urea filtration decreases

B. Urea absorption decreases

C. Urea filtration increases

D. Urea secretion increases

188. The semi-permeable membrane of the dialysismachine functions in a manner most analogous towhich part of the kidney?

A. Glomerulus

B. Ureter

C. Descending loop of Henle endothelium

D. Vasa recta

67

GO ON TO THE NEXT PAGE.

03 MCAT FL Test3 06/27/2003 10:27 AM Page 67

Page 68: Full length 3

189. A patient with renal failure has nephrons which lackthe ability to actively secrete or reabsorb any sub-stances. Which of the following actions will thepatient’s kidney still be able to perform?

A. Removal of salt from the blood

B. Production of hypertonic urine

C. Production of hypotonic urine

D. Conservation of amino acids

190. All of the following are removed from the plasma bydialysis EXCEPT:

A. Na+

B. K+

C. Urea

D. Glucose

191. Healthy kidneys secrete the hormone renin inresponse to decreased arterial pressure. Renin secre-tion leads to aldosterone secretion by the adrenal cor-tex. The body would most likely respond to ingestionof a large volume of isotonic solution by:

A. decreasing renin secretion and increasingsodium and water reabsorption.

B. decreasing renin secretion and decreasingsodium and water reabsorption.

C. increasing renin secretion and increasingsodium and water reabsorption.

D. increasing renin secretion and decreasingsodium and water reabsorption.

68

GO ON TO THE NEXT PAGE.

03 MCAT FL Test3 06/27/2003 10:27 AM Page 68

Page 69: Full length 3

192. Concentrated nitric acid is capable of autoionizationaccording to:

2 HNO3 H2NO3+ + NO3

-

H2NO3+ H2O + NO2

+

One piece of experimental evidence for this phe-nomenon is that certain activated aromatic com-pounds have been found to undergo nitration inconcentrated nitric acid without the assistance of acatalyst. Which of the following compounds is mostlikely to undergo such an unassisted nitration?

A.

B.

C.

D.

193. Which of the following reactants should be used toform the product shown below?

A. Sodium ethanoate

B. 1-propanol

C. Ammonia and ethane

D. Dimethyl ether

194. Oversecretion of gastric HCl can be treated by sever-ing the vagus nerve in a procedure called a vagotomywhich reduces parasympathetic activity. Which ofthe following effects is LEAST likely to be caused bya vagotomy?

A. Decreased heart rate

B. Decreased gastric motility

C. Decreased HCl production

D. Increased blood pressure

195. Tetrodotoxin, the extremely potent poison producedby the puffer (fugu) fish, binds tightly to Na+ chan-nels and blocks the flow of Na+ ions but does notaffect K+ or Cl– channels. Tetrodotoxin directlyblocks which phase of action potential propagation?

A. Depolarization

B. Repolarization

C. Hyperpolarization

D. Saltatory conduction

196. Female athletes may fail to menstruate (amenorrhea)because extremely low body fat reduces the releaseof GnRH from the hypothalamus, affecting the secre-tion of LH and FSH. Which of the following is mostlikely to be absent in a female long-distance runner?

A. Ovary

B. Anterior pituitary

C. Corpus luteum

D. Hypothalamus

O

ClH3C

O

OH3C

?

OCH3

OCH3

N

Cl

O

O

Cl Cl

CN

F

Questions 192 through 196 are NOT basedon a descriptive passage.

69

GO ON TO THE NEXT PAGE.

03 MCAT FL Test3 06/27/2003 10:27 AM Page 69

Page 70: Full length 3

Passage IX (Questions 197–201)

Sugars are carbohydrates, that is, molecules usuallywith the empirical formula C(H2O), and structural formu-las made up of polyhydroxy aldehydes or ketones. Becauseof their polyfunctional nature, sugars can undergo a widevariety of transformations upon treatment with acids,bases, or heat, and upon reaction with other simplereagents and enzymes. While many sugars occur in natureand are thus readily available, the synthesis and modifica-tion of simple sugars is a necessary step in studies of enzy-matic processes.

Higher sugars can be synthesized from the simple car-bohydrate D-glyceraldehyde with the following procedure:

D-glyceraldehyde (Compound A) is reacted with HCNto produce a cyanohydrin (Compound B). Compound B isthen treated with hydrogen gas and a modified palladiumcatalyst (similar to the Lindlar reagent) to give CompoundC. Compound C is hydrolyzed to give the higher sugars inMixture D. This reaction is summarized in Figure 1. Mix-ture D contains two compounds, which can be separatedby crystallization. Two doublets near 9.5 (δ, ppm) areobserved in the 1H NMR spectrum of mixture D, with eachdoublet corresponding to one of the two products presentin the mixture. IR spectroscopy shows broad absorptionsfor both products around 3300 cm–1.

Figure 1 Synthesis of higher sugar

The hydroxyl groups of carbohydrates can also partic-ipate in reactions. For example, D-glyceraldehyde canreact with chloromethane under basic conditions to yield acompletely methylated product. This SN2 reaction isshown in Figure 2.

Figure 2 Methylation of D-glyceraldehyde

197. The reaction shown in Figure 2 most likely proceedsvia the formation of a(n):

A. water leaving group.

B. nucleophilic alkoxide ion.

C. carbocation intermediate.

D. tetrahedral intermediate.

198. The modified palladium catalyst is used instead ofplatinum to:

A. avoid reduction of the cyanohydrin to theamine.

B. make the reaction energetically favorable.

C. act as a Lewis acid.

D. reduce the likelihood of attack at the carbonylcarbon.

199. The two sugars in Mixture D are:

A. enantiomers.

B. anomers.

C. epimers.

D. disaccharides.

HC

H

CH2OH

OH

O

D-glyceraldehyde

2 CH3Cl, NaOHHC

H

CH2OCH3

OCH3

O

Compound E

HC

H

CH2OH

OH

O

D-glyceraldehyde(Compound A)

HCN cyanohydrin(Compound B)

H2, Pd-BaSO4,

H+, H2O, 4atm

Compound C

higher sugar(Mixture D)

H2O

CHOH

H

CH2OH

OH

HC NH

70

GO ON TO THE NEXT PAGE.

03 MCAT FL Test3 06/27/2003 10:27 AM Page 70

Page 71: Full length 3

200. What are the missing compounds, respectively, in thefollowing reaction?

A.

B.

C.

D.

201. In glucose, the carbonyl carbon can be attacked,intramolecularly, by the hydroxyl oxygen of carbon-5 to form:

A. glucofuranose.

B. a hemiacetal.

C. a lactone.

D. a glycoside.

HC

H

CH2OCH3

OCH3

NNH2 CH3

H

CH2OCH3

OCH3

,

OH

H NH2

H OCH3

CH2OCH3

O-

H NH2

H OCH3

CH2OCH3,

HC

H

CH2OCH3

OCH3

NNH2 HC

H

CH2O-

O-

NNH2

,

HC

H

CH2ONH2

ONH2

O HC

H

CH2ONH2

ONH2

O

,

Compound EH2NNH2

?NaOH

?

71

GO ON TO THE NEXT PAGE.

03 MCAT FL Test3 06/27/2003 10:27 AM Page 71

Page 72: Full length 3

Passage X (Questions 202–208)

Before birth, the rodent brain is sexually undifferenti-ated. It is only in the first few days following birth, during aperiod referred to as the critical period, that the rodent braindifferentiates along male or female lines. The hormonetestosterone plays a critical role in this development. Specif-ically, sexual differentiation is determined by the presence ofestradiol, an estrogen derivative of testosterone, in certainareas of the brain. Testosterone is converted to estradiol incritical brain cells that contain the enzyme aromatase.

To study the effects of testosterone on the neonatalrodent brain, the following experiments were conducted:

Figure 1

The above research, combined with additional studies,concluded that testosterone has two “organizational”effects on the male rodent brain:

Defeminization

Moderate levels of testosterone-derived estradiol dur-ing the critical period are sufficient for defeminization ofthe brain. Defeminization of the rodent brain results in lossof estrogen positive feedback on LH and FSH secretionand the ensuing loss of cyclicity, as well as loss of femalesex behavior.

Masculinization

High levels of estradiol due to high levels of testos-terone during the critical period results in masculinizationof the brain. Masculinization leads to the induction of malesex behavior including antagonism towards other malesand the mounting of females.

202. A mutant male neonatal rodent with a defective aro-matase enzyme is injected with large doses of testos-terone during the critical period. Which of thefollowing would occur?

A. Induction of male sex behavior

B. Development of female sex behavior

C. Loss of both cyclicity and female sex behavior

D. Loss of both cyclicity and female sex behaviorand induction of male sex behavior

203. The conversion of testosterone to estradiol is whattype of reaction?

A. Reduction

B. Aromatization

C. Electrophilic aromatic substitution

D. Methylation

204. A researcher proposes that very low doses of estra-diol are required for induction of female sex behav-ior and cyclicity in the rodent brain. Which of thefollowing observations would best support thishypothesis?

A. Female neonates injected with the anti-estrogentamoxifen are acyclic and show neither male norfemale sex behavior.

B. Female neonates lacking the aromatase enzymedevelop normally.

C. Female neonates injected with large doses ofestradiol are acyclic and show male sex behav-ior.

D. Male neonates injected with low dosages ofestradiol develop normally.

O

CH3

CH3

OH

HO

CH3

OH

testosterone

estradiol

neonatal malecastrated atbirth

wait 2-3 months

transplantovaries cycles and shows female

sex behavior

Expt.

2

3neonatal malecastrated atbirth

inject testosterone at birth

transplantovaries acyclic and shows

male sex behavior

neonatal femalecastrated atbirth

transplantovaries acyclic and shows

male sex behavior

inject testosterone at birth

4

female neonatein critical period

acyclic and shows male sex behavior

inject large dosage of estradiol

5

neonatal male

wait 2-3 months

transplantovaries acyclic and shows

male sex behavior1

72

GO ON TO THE NEXT PAGE.

03 MCAT FL Test3 06/27/2003 10:27 AM Page 72

Page 73: Full length 3

205. In Experiment 2, researchers most likely waited 2-3months before implanting ovaries:

A. to allow the rat to recover from previoussurgery.

B. to allow testosterone levels to rise to necessaryconcentrations.

C. to wait for the critical period to pass.

D. to promote the defeminization of the rat brain.

206. An adult male rat that is acyclic is observed to mountfemales. However, the rat also allows itself to bemounted by male rats. According to the passage, themost likely explanation for these observations is that:

A. the rat is hermaphroditic, with both male andfemale sex organs.

B. the rat has been demasculinized.

C. the rat has been masculinized but not fullydefeminized.

D. the rat has been defeminized but not fully mas-culinized.

207. Steroid hormones bind to receptors in the nucleusand directly regulate:

A. the transcription of mRNA.

B. the translation of protein.

C. the production of cAMP.

D. the replication of DNA.

208. A normal female rat injected with estradiol willrespond by:

A. decreasing secretion of LH from the anteriorpituitary.

B. increasing secretion of LH from the anteriorpituitary.

C. decreasing secretion of GnRH from thehypothalamus.

D. increasing secretion of progesterone from thefollicle.

73

GO ON TO THE NEXT PAGE.

03 MCAT FL Test3 06/27/2003 10:27 AM Page 73

Page 74: Full length 3

209. Maternal lineage can be traced by sequencing themitochondrial DNA because the mitochondrialgenome is derived primarily from the mother. Thebest explanation for this phenomenon is that:

A. sperm have no mitochondria and thus cannotcontribute to the mitochondrial genome of theoffspring.

B. the zygote divides rapidly, diffusing the paternalmitochondria amongst many cells.

C. following penetration, the sperm derived mito-chondria disintegrate within the egg.

D. all the genes coding for mitochondria arelocated on the X chromosome.

210. The endosymbiosis theory proposes that mitochon-dria originated through the engulfing of aerobicprokaryotes by anaerobic host prokaryotes. Many ofthe enzymes required for aerobic respiration arelocated on the inner membrane of the mitochondria.Where would these enzymes, or their precursors,most likely have been found three billion years ago(prior to endosymbiosis)?

A. In the cytoplasm of the anaerobic prokaryotichost cells

B. On the plasma membrane of the anaerobicprokaryotic host cells

C. On the plasma membrane of the aerobicprokaryotic engulfed cells

D. In the cytoplasm of the prokaryotic aerobic cells

211. A mountain climber living at sea level ascends to avery high altitude during the course of a day longclimb. By the end of the day, all of the followingacclimatizations will occur EXCEPT:

A. increased tidal volume.

B. increased respiration rate.

C. right shift of the hemoglobin dissociation curve.

D. increased concentration of erythropoietin in theblood.

212. At its isoelectric point, an amino acid in an electricfield will migrate towards the:

A. anode.

B. cathode.

C. basic region.

D. it will not migrate.

213. Which of the following species will form the moststable radical?

A. C.

B. D.

214. A hydropathy plot indicates the hydrophilicity (nega-tive values) and hydrophobicity (positive values) ofdifferent regions of a protein. According to thehydropathy plot below, how many cellular membranespanning regions are possible in this protein?

A. 1

B. 2

C. 3

D. 4

STOP. IF YOU FINISH BEFORE TIME IS CALLED,CHECK YOUR WORK.YOU MAY GO BACK TO ANYQUESTION IN THIS SECTION ONLY.

+40

+20

0

-20

-40

0 20 40 60 80 100

amino acid residues

hydr

opat

hy in

dex

Questions 209 through 214 are NOT basedon a descriptive passage.

74

03 MCAT FL Test3 06/27/2003 10:27 AM Page 74

Page 75: Full length 3

03 MCAT FL Test3 06/27/2003 10:27 AM Page 75